Középiskolai Matematikai és Fizikai Lapok
Informatika rovattal
Kiadja a MATFUND Alapítvány
Már regisztráltál?
Új vendég vagy?

Fórum: GEOMETRIA

  [1]    [2]    [3]    [4]    [5]    [6]    [7]    [8]    [9]    [10]  

Szeretnél hozzászólni? Jelentkezz be.
[467] rizsesz2006-09-05 09:41:19

Egy újabb remek feladat: adott egy kör alapú henger alakú tortánk, és ezt kellene 12 vágással 80 szeletre (részre) vágni. Semelyik 3 vágás nem mehet át egy ponton, viszont bármelyik kettőnek van közös metszéspontja.

[466] Csimby2006-08-29 20:12:49

Nálam cd a kör átmérője volt, nálad gondolom a sugara, mert úgy kijön amit írsz. Köszönöm, tetszik!

Előzmény: [465] jonas, 2006-08-29 00:40:45
[465] jonas2006-08-29 00:40:45

Bocs, a 77. feladatot tényleg nem intézi el. Nos, c=1 triviálisan jó.

Az éles határ c-re  1/\sqrt 3 . Ennek a bizonyítása a Reiman: Geometria és határterületei könyv 15.3. (Helly tételes) fejezetében van benne. A bizonyítás a következőn múlik. Először belátod, hogy a korlát jó háromszögekre (szabályos háromszögre pont  1/\sqrt 3 az arány), aztán veszed a ponthalmaz minden három pontjára azon pontok halmazát, amelyektől mindhárom pont legfeljebb  d/\sqrt 3 távolságra van, majd erre a halmazrendszerre alkalmazod a Helly-tételt.

Előzmény: [462] Csimby, 2006-08-28 14:46:02
[464] Csimby2006-08-28 22:31:19

Köszi! Ez 76.-ot tényleg elintézi, de 77.-et szerintem nem.

Előzmény: [463] jonas, 2006-08-28 19:01:13
[463] jonas2006-08-28 19:01:13

76., 77. feladatokra. Ha a töröttvonal egy nagyon lapos egyenlőszárú tompaszögű háromszög, akkor a kört csak egyféleképpen lehet kiválasztani, és a háromszög kerületéhez képest ez akármilyen nagy lehet. Tehár nem igaz az állítás.

Előzmény: [462] Csimby, 2006-08-28 14:46:02
[462] Csimby2006-08-28 14:46:02

76. feladat

Adott egy h hosszú zárt töröttvonal. igaz-e hogy mindig kiválasztható 3 csúcsa, melyek köréírható köre lefedi az alakzatot és átmérője kisebb/egyenlő mint h/2 (ha nem, akkor mi a legkisebb c konstans amit az \frac{1}{2} helyére írhatunk).

77. feladat

Van e olyan c konstans, hogy bármely d átmérőjű alakzat lefedhető egy cd átmérőjű körrel. (Mi a legkisebb ilyen c)

Nem tudom milyen nehezek, csak eszembe jutottak.

[461] rizsesz2006-08-21 22:24:26

elnéztem a feladatot :) 9 síkunk van, és 74 a kérdéses limit, ami pont passzol :)

[460] jonas2006-08-20 19:38:12

Na nézzük. Akkor most át is gondolom a választ, nem csak tippelek.

Ha lerakjuk az első síkot, akkor utána már csak azt kell megnézni, hány részre osztja a két félteret a maradék hét sík. Ez ugyanannyi, mint ahány részre hét egyenes osztja a síkot, csak kétszer kell számolni. Mármost erre viszont tudjuk a választ, mégpedig 1+(1+2+...+7)=29 síkrész, így aztán összesen 2.29=58 térrész keletkezik.

Előzmény: [459] 2501, 2006-08-20 17:26:04
[459] 25012006-08-20 17:26:04

Ha ez a megoldás jó, akkor azt is jelenti, hogy minden n-hez csak egyetlen F tartozik, tehát az a bizonyos maximum egyben minimum is. :)

Előzmény: [458] 2501, 2006-08-20 14:49:21
[458] 25012006-08-20 14:49:21

Szerintem feltehetjük úgy is a kérdést, hogy egy gömb 8 főköre, amelyek közül semelyik háromnak nincs közös pontja, maximum hány részre osztja a gömböt.

Ha a főkörök száma n, akkor a metszéspontok száma n(n-1), mivel minden párnak két metszéspontja van. Minden pontból 4 darab főkör-szegmens indul ki (és minden szegmensen két pont osztozik), tehát a szegmensek száma a csúcsok számának kétszerese, 2n(n-1).

Mivel az így létrejövő gráf gömbre rajzolható :), alkalmazható rá az Euler-féle poliédertétel:

V + F - E = 2

F = 2 + E - V

F = 2 + 2n(n-1) - n(n-1)

F = 2 + n(n-1)

Pl:. 3 főkör esetén V = 3*2, E = 2*3*2, F = 2 + 3*2, ami egész jól egybevág azzal, amit az oktaéderről tudunk. :)

Ez a gondolatmenet n = 8 esetében 58-at ad meg maximumként, ami nekem őszintén szólva kicsit soknak tűnik. :) Ha valaki megtalálja benne a hibát, legyen szíves, szóljon! Köszönöm.

Előzmény: [456] rizsesz, 2006-08-19 14:51:47
[457] jonas2006-08-20 12:45:31

Szerintem 37-re.

Előzmény: [456] rizsesz, 2006-08-19 14:51:47
[456] rizsesz2006-08-19 14:51:47

Sziasztok! Lenne egy feladatom:

Adott a térben egy pont, és 8 olyan sík, amelyek mindegyike átmegy ezen a ponton, viszont semelyik 3 sík nem megy át egy egyenesen. Maximálisan hány részre oszthatják a teret ezek a síkok? (Az eredeti feladatban az a kérdés, hogy 36 részre oszhatják-e).

[453] mephisoft2006-08-08 00:45:43

Köszi! Azt azért gondolhatnátok, hogy aki ilyen egyszerűt kérdez, az nem biztos, hogy megérti az ilyen (nekem) magasröptű válaszokat. - Viszont időközben összeszedtem magam, fölírtam a két ponton átmenő egyenes egyenletét, az erre merőleges, a harmadik ponton átmenő egyenes egyenletét, ezt megoldottam, mint egyenletrendszert, az kiadja a metszéspontot, végül arra is rájöttem, hogy kell két pont távolságát kiszámolni (Pithagorasz :) Már meg is írtam ebből a Java kódot, de a kipróbálása már holnapra marad ...

Azért még egyszer köszönöm, jó tudni, hogy vannak segítőkész emberek, akik ráadásul még a matekhoz is értenek.

[455] xviktor2006-08-07 23:38:20

Hali!

Egy masik lehetseges modszer. Legyen A es B az egyenesen C pedig a harmadik pont. A 3 pont meghataroz egy haromszoget, a keresett tavolsag, az AB oldalhoz tartozo magassag. A haromszog teruletet kiszamoljuk eloszor Heron-keplettel, majd a szokasos keplettel: \sqrt{s\cdot (s-AB)\cdot (s-AC)\cdot (s-BC)}=\frac{AB\cdot m_{AB}}2

Nem mondom, hogy egyszerubb, mint a masik megoldas, de pont ez a szep a matekban, hogy sok megoldasa van egy feladatnak.

Udv: Vik

Előzmény: [451] mephisoft, 2006-08-07 21:27:54
[452] 25012006-08-07 22:35:16

Egy P pont "előjeles távolsága"* egy hipersíktól \frac{\vec N \cdot \vec R}{|\vec N|}, ahol \vec N a hipersík egy tetszőleges normálvektora, \vec R pedig a hipersík egy tetszőleges pontjából P-be mutató vektor.

Jelen esetben a hipersík egy egyszerű egyenes. :) Alkalmas \vec N és \vec R pl.: \matrix{N_x = A_y-B_y\cr N_y = B_x-A_x}, \vec R = \vec P - \vec A, ahol A és B az egyenes két pontja.

*A normál irányával ellentétes oldalon negatív.

Előzmény: [451] mephisoft, 2006-08-07 21:27:54
[451] mephisoft2006-08-07 21:27:54

Tudna nekem valaki segíteni ... Tudtam én ezt valamikor, de elfelejtettem ...

Egy síkban adott egy két pontjával meghatározott egyenes, valamint egy harmadik pont. Hogyan tudom a pont és az egyenes távolságát kiszámolni?

Köszi

[450] Károly2006-07-19 08:58:51

Nem az ilyeneknek az elterjedt neve ez. (És ez nem is "geometria", azaz a kérdés egy kicsit off.)

Háromszögfüggvényeknek az olyan függvényeket hívják, mint pl. az |x|\le1 intervallumon értelmezett 1-|x| függvényt - és ennek periodikus ismétlődéseit és egyéb transzformáltjait (ilyen-olyan nyújtások stb.).

Ami Téged érdekel, az valószínűleg a "metrikus terek leképezései" avagy a "topológia" tárgyszó alatt található.

Üdv

K.

Előzmény: [444] epsilon, 2006-07-12 08:38:35
[449] Érdeklődő2006-07-16 12:15:13

Tovább bővíteném a kérdéseim körét. Van-e magyar nevük (ha van, akkor mi az?) az Arkhimédészi, katalán, Johnson féle testeknek? Találtam pár weboldalt, de ott csak angol neveket találtam. Létezik olyan magyar nyelvű könyv, amiben ezek a testek összeszedve magyar nevekkel és jellemzőkkel le vannak írva??? (Ez 5(szabályos)+13+13+92=123 test) Aki tud, kérem segítsen!!!

[448] Érdeklődő2006-07-14 17:47:56

Megnéztem a félig szabályos testeket, de nem teljesen világos minden számomra. A következőknek nem találtam meg a magyar megfelelőjét:

Cuboctahedron, Rhombicuboctahedron,

Saját elgondolás szerint hasonlóan a többihez tudnám "magyarítani", de nem tudom helyes lenne-e. Ezeknek mi a magyar megfelelője?

[447] Érdeklődő2006-07-13 21:25:55

Köszönöm a segítséget!!!

[446] Lóczi Lajos2006-07-13 16:38:44

Annak idején keresgéltem ezeket a neveket, amikor magyarítanunk kellett őket; egy párat l. a thesaurus.maths.org fogalomtárban.

Itt meg egy animáció is van róluk:

http://www.jgytf.u-szeged.hu/tanszek/matematika/polieder/Arkhimedesz/nevek.gif

Előzmény: [445] Érdeklődő, 2006-07-12 21:33:18
[445] Érdeklődő2006-07-12 21:33:18

Sziasztok! Az iránt érdeklődöm, hogy az Arkhimedeszi testeknek (félig szabályos) van-e magyar nevük?

[444] epsilon2006-07-12 08:38:35

Üdvözlök Mindenkit! Az lenne a kérdésem, hogy tud-e Valaki, akár nemzetközi nyelven elérhető forrásanyagot az ú.n. HÁROMSZÖGFÜGGVÉNYEKRŐL? (Leegyszerüsítve, egy f függvényt 3-szögfüggvénynek neveznek, ha a 3-szög egyenlőtlenséget teljesítő három a, b, c számra a függvény képeire is fennál a 3 db 3-szög egyenlőtlenség. (Pl. a (konkáv és monoton növekvő és f(0)=0) függvények ilyenek). Előre is köszönöm! Üdv: epsilon

[443] Hajba Károly2006-07-10 12:27:33

A CAD programokkal, így az AutoCAD-del is lehet egynéhány dolgot csinálni a felsoroltak közül, de nem kimondottan a geometriai oktatás szemléltetés céljára, hanem a mérnöki termék szakmában szokásos bemutatésára. Axonometria, perspektíva, vágás mindben van, ahol van 3D. Az régebbi AutoCAD-nek is van ilyen kiegészítője, habár az alapvetően 2D-s.

Ha a 3D-s programot nem kimondottan ábrázoló geometriára hegyezték ki, akkor a kivánt feladatokat legfeljebb csak közvetve tudod elérni, a program készletében megtalálható elemi lépések sorozatával. E mellett pedig rengeteg számodra felesleges lehetőséget találsz benne, ami a megcélzott területben való dolgozáshoz elengedhetetlen.

Így feltehetőleg neked egy olyan általános 3D-s program kellene, amiben egyes szerkesztési lépések sorozata programozható.

Talán a következők lehetnek számodra megfelelők. Én nem ismerem ezeket, csak hallottam róluk: OpenGL és VRML Azaz olyan rendszereket keress, amik ezen eljárásokat ismerik. Az OpenGL egy térbeli ábrázolási eljárás és szerkesztés, amit az adott 3D-s program szabványosan alkalmazhat.

http://www.inf.u-szeged.hu/oktatas/jegyzetek/KubaAttila/opengl/starthu.xml

Előzmény: [439] matspec, 2006-07-10 00:17:26
[442] LENSZ2006-07-10 00:52:30

erre tellik :P

Előzmény: [441] matspec, 2006-07-10 00:51:10
[441] matspec2006-07-10 00:51:10

Ez aztán segítség... :P

[440] LENSZ2006-07-10 00:49:45

én tudok neked mondani!:D

körző ceruza 2.0 :O

Előzmény: [439] matspec, 2006-07-10 00:17:26
[439] matspec2006-07-10 00:17:26

Sziasztok! Most járok itt először... :) Szeretnék segítséget kérni tőletek. Olyan ábrázoló geometriás programra lenne szükségem, amiben lehetőleg minden fontos benne van: Monge-féle merőleges vetítés, axonometria, perspektíva, lehet benne testek áthatását vizsgálni, kúpot vagdosni, ilyesmi. AutoCAD-del kísérleteztem, nem sok sikerrel... Előre is köszönöm, ha valaki tud néhányat mondani! :)

[438] Lóczi Lajos2006-06-11 22:08:53

A cikkben szereplő metszetábrákon (vagy egy kicsit nagyobb ábrákon itt) a metszősíkot önmagával párhuzamosan told bele a kúp csúcsába, és akkor érinti / kimetszi azt az 1 vagy 2 alkotót, amiket keresel.

Előzmény: [437] god, 2006-06-11 11:46:47
[437] god2006-06-11 11:46:47

Üdvözlet!

Kós Rita a "Kúpszeletek és Dandelin-gömbjeik" c. cikkében azt írja, hogy ha egy egyenes kúpot a csúcsán át nem menő síkkal elmetszünk, akkor vagy kört, parabolát, ellipszist v hiperbolát kapunk. Olyan feltételeket ír pl. hogy 2 alkotóval párhuzamos. Mit jelent az, hogy egy sík 1, 2 alkotóval párhuzamos? Illetve mi az az 1, 2 alkotó amivel párhuzamos? Én nem látom...

Előre is köszönöm!

[436] epsilon2006-06-10 10:04:38

Kösz, kiindulási ötletnek nem rosz! ;-)

[435] jonas2006-06-09 22:53:35

Indulj ki abból, hogy a négy sarokhoz négy különböző négyzetnek kell tartoznia, és azon kívül csak egy négyzeted van. Innentől három lehetőség van:

A E B
C D
A B
C E D
A B
C D
D

Az első kettőnél a nagy téglalap nem lehet négyzet, a harmadiknál a kis téglalapok. Persze ez nem precíz bizonyítás.

Előzmény: [434] epsilon, 2006-06-09 22:21:08
[434] epsilon2006-06-09 22:21:08

Igen, ismerem a könyvet, nagyon kedves,és tanulságos! Sok helyen kerestem az 5-re egy szimpatikus bizonyítást, most meg az is felbuzdított, hogy a Kengurú egyik selejtezőjén feltették a kérdést, hogy hány darabra nem darabolható egy négyzet, és persze ott volt az 5 is, ez azt a gyanút keltette bennem, hogy ha ilyen helyen tippelés formájában adták föl, akkor csak kell lennie valamilyen egyszerű bizonyításnak! Hátha valaki találkozott ilyennel?!

[433] jonas2006-06-09 21:42:58

Igen, akárhány négyzetre feldarabolható. A bizonyítás megtalálható Reiman István könyvének 15. fejezetében. Ez azon áll, hogy bármely darabolásból kaphatsz egy 3-mal több négyzetté darabolást, ha az egyik kis négyzetet négy felé osztod, így elég megmutatni, hogy a négyzetet 4, 6, és 8 felé lehet vágni. Az 5 négyzetes esetre nem ad egyszerű bizonyítást.

Előzmény: [432] epsilon, 2006-06-09 19:41:05
[432] epsilon2006-06-09 19:41:05

Helló! Nagyon szimpatikus feladat a négyzetnek az akárhány négyzetre darabolása, ez az "akárhány" természetesen 4 vagy több mint 5. Régóta felmerült bennem a kérdés, hogy van-e egyszerű bizonyítás arra, hogy egy négyzet nem darabolható fel 5 darab négyzetre? Ha van véleményetek, bizonyításotok szívesen várom! Üdv: epsilon

[431] Csimby2006-06-05 01:23:39

Szabó László: Konvex Geometria (Egyetemi jegyzet) Az ELTE TTK Déli épületében a 4. emeleten a geometria tanszék titkárságán lehet kapni, ha jól emlékszem kb. 500 ft.

Előzmény: [430] tyuki, 2006-06-03 17:18:03
[430] tyuki2006-06-03 17:18:03

Szeretnélek bennetek megkérni arra, ha tudtok valami jó web-címet, amin találhatok összefüggő (hosszú) szöveget az affi-geometriáról, küldjétek el légyszi az e-mail címemre: nytuki@orangeportal.sk. Ha valami jó könyvet is tudtok ajánlani, azt is megköszönném. Köszi

[428] Csimby2006-05-09 19:52:25

Igen, a nyíl ezt próbálta szimbólizálni :-)

Kérdés: ha a fő járat nem egyirányú, akkor vajon minden permutáció előáll?

Előzmény: [427] jonas, 2006-05-08 23:51:01
[427] jonas2006-05-08 23:51:01

Igen, feltéve, ha a fő járat egyirányú.

Előzmény: [426] Csimby, 2006-05-08 23:32:51
[426] Csimby2006-05-08 23:32:51

Vagy inkább annyiféleképpen, ahányféle sorrendben n-2 db. hangya ki tud jönni egy járatból, ha a járat közepén található egy zsákutca (amiben tetszőlegesen sok hangya elfér de nem fér el egyszerre kettő menetiránnyal szemben) :-)

(Mind az n-2 db. hangyának ki kell jönni)

Előzmény: [425] jonas, 2006-05-08 23:05:06
[425] jonas2006-05-08 23:05:06

Hát igen, ehhez készült az ábra eredetileg.

Ugyanannyiféleképpen, mint ahány n-2 darab egyesből és n-2 darab mínusz egyesből álló sorozat van, aminek minden részletösszege pozitív. A bizonyítás levezethető az eredeti ábra postscript forrásából.

Ezzel, remélem, még nem árulok el sokat.

Elmondom majd a bizonyítást, ha érdekel valakit.

Előzmény: [423] Csimby, 2006-05-08 21:49:32
[424] Csimby2006-05-08 21:52:04

Sorry, legyen inkább az előző a 74. feladat :-)

Előzmény: [423] Csimby, 2006-05-08 21:49:32
[423] Csimby2006-05-08 21:49:32

75. feladat Egy szabályos n-szöget átlóival n-2 db. háromszögre daraboltunk. Hányféleképpen tehetjük ezt meg?

[422] jonas2006-05-08 21:15:22

Lehet, hogy sokan ismerik a következő feladatot.

73. feladat. Ha egy 2n+1 oldalú szabályos sokszöget 2n-2 átlóval háromszögekre bontunk, akkor milyen határok között változhat a keletkező háromszögek közül a hegyesszögűek száma?

Segítségül itt egy ábra.

[421] Vonka Vilmos Úr2006-05-02 09:43:54

Ez egy érdekes probléma. Nekem az a gyanúm, hogy ezen a projektív geometria nem tud segíteni. Ha ugyanis megadjuk pl az A, B, C, D pontokat, akkor ahogy kísérletezgettem GeoGebrában, úgy látom, előfordulhat, hogy akár 3 olyan ellipszis is van, aminek ezek pontjai, és A valamelyik tengely végpontja. Ha pedig 3 vagy több megoldás van, akkor hiába is keresünk jó szerkesztési eljárást. Persze az, hogy A a nagytengely, és nem a kistengely végpontja, az egy további szűkítés - ezt azonban projektív módon megfogni nehézkes, legfeljebb úgy tudom elképzelni, hogy valahogyan azt próbáljuk meg kihasználni, hogy a valós fókuszok egyenesén van rajta. Nekem ez sajnos nem sikerült.

Érdekes azonban, hogy ha A, B, C, D egy körön van, akkor már létezik nagyon egyszerű szerkesztési eljárás a tengelyek irányára: projektív eszközökkel ugyanis meg lehet mutatni, hogy akkor az AB, CD egyenesek szögfelezői párhuzamosak lesznek a keresett ellipszis tengelyeivel.

Előzmény: [418] Morci, 2006-04-26 12:00:31
[420] Vonka Vilmos Úr2006-05-02 09:34:08

Ha az ellipszis 5 pontját ismerjük, akkor projektív módon projektív sugársorok metszési alakzataként tudjuk előállítani. A sugársorok közötti projektív leképezést három megfelelő elempár határozza meg, így ha kiválasztjuk az adott A és B pontokat, mint tartópontokat, akkor az AC, AD, AE egyeneseknek megfelelő BC, BD, BE egyenesekkel megadtunk egy, a kúpszeletet meghatározó projektivitást. Ha a sugársorokat metszük az adott (e) egyenessel, akkor az e egyenesen projektív pontosorokat kapunk. Ebben a projektivitásban az önmagának megfelelő pont adja az egyenes és a kúpszelet metszéspontját. Így erre a projektivitásra a Steiner-féle kettőselem-szerkesztést alkalmazva kapjuk meg a keresett metszéspontokat. A Steiner-féle kettőselem-szerkesztésnél a síkon felvett tetszőleges körre annak egy tetszőleges pontjából rávetítjük a projektivitás megfelelő pontpárjait. Kiválasztva (az ábrán például) a C1' és C2' pontokat, rendre a C2', D2', E3' és C1', D1', E1' pontokat vetítve belőlük, perspektív sugársorokat kapunk. Ezen perspektivitás tengelye a kört a P1', P2' pontokban metszi, ezeket visszavetítve az (e) egyenesre, megkapjuk a keresett kettőspontokat.

Előzmény: [419] HoA, 2006-05-02 08:55:07
[419] HoA2006-05-02 08:55:07

Milyen Steiner-szerkesztésre gondolsz? Ezen a néven nem találtam olyan szerkesztést, mely egy pontjaival adott ellipszis és egy egyenes metszéspontjait adná meg.

Előzmény: [407] Vonka Vilmos Úr, 2006-04-07 11:25:57
[418] Morci2006-04-26 12:00:31

Köszönöm a segítséget.

Átgondolom, kipróbálom, lehet lesz még kérdésem... nem ismerem mindegyik dolgot amit leírt, de rákeresgetek. (Nem matematikával-geometriával foglalkozom alapvetően.)

A probléma abban módosult, hogy kiderült nem 5 pont, hanem négy pont alapján kellene előállítani az ellipszist. A pontok közül 1 db az speciális azaz a nagytengely egyik végpontja. a többi 3 darab teljesen általános. Ebben kérnék segítséget. Próbáltam már kérdezgetni több felé, elvileg ez csak számítással oldható meg?

Üdv. Morci

Előzmény: [407] Vonka Vilmos Úr, 2006-04-07 11:25:57
[417] Sirpi2006-04-17 00:53:23

Szerintem érdemes próbálkoznod ebben a topikban. A külön témát, amit nyitottál, töröltem, a linknél teljesen jó helye van a kérdésednek.

Előzmény: [416] kenez, 2006-04-16 00:46:57
[416] kenez2006-04-16 00:46:57

Az informatikus kollégáktól kérnék segítséget. A probléma a következő. Van egy kockám, és szeretném azt megforgatni egy programmal. 1. kérdés : hogyan számítom ki a 8 csúcspont koordinátáit, ha elforgatom őket x,y,z tengely körül, és mi ennek a matematikai alapja(csak hogy értsem is hogy megy!), 2. kérdés : 3D - 2D leképezés módszere érdekelne nagyon részletesen. Ja és még valami. Van még ingyen letölthető szerkesztőprogi az Euklidesen kívül? Köszi mindenkinek! Kenéz

[415] kenez2006-04-16 00:20:28

Kösz, HoA, látod, néha még egy mondat is csodákra képes.... A helyzet az, hogy ebbe valóban nem gondoltam bele, mivel a szerkesztésnél annyira evidens volt, és látható, hogy nem gondoltam, hogy bizonyítani kéne. Mivel látványos volt a dolog, abba se gondoltam bele, csak akkor lehet trapéz, ha van egy párhuzamos oldalpárja. Hát nincs is neki. Ennyit erről. Kicsit jobban figyelni(Ezt magamnak mondom!).

[414] HoA2006-04-15 22:45:14

Ábra (igaz, fejjel lefelé) mellékelve. Segítség: Miből gondolod, hogy pl. ABED húrtrapéz? AE és BD párhuzamosságát bizonyítani kellene.

Előzmény: [412] kenez, 2006-04-15 18:20:50
[413] kenez2006-04-15 18:27:05

Az ábrát elfelejtettem feltenni!

[412] kenez2006-04-15 18:20:50

Hello mindenkinek! Megoldottam egy feladatot, mindjárt le is írom a szövegét, a megoldásom viszont nem volt jó, nagyon szeretném, ha valaki megcáfolná az én megoldásomat, megmondaná, miért nem helyes. Köszönettel: Kenéz A feladat: Középkori építészek használták a következő szerkesztést a szabályos ötszög előállítására: Rajzoltak egy négyzetet, aztán megrajzolták a négyzet oldalaival párhuzamos szimmetriatengelyeket, majd a négyzet bele- és köréírt körét. A négyzet A oldalfelező pontját összekötötték a körülírt kör PQ átmérőjének végpontjaival. A négyzet beírt körét az AP és PQ szakaszok a B és E pontokban, a négyzet alsó felét adó KLMN téglalap átlói pedig a C és a D pontokban metszik. Az ABCDE pontok alkotják az ötszöget. Valóban szabályos ez az ötszög?

Az én megoldásom: Behúzva az AD BE CE BD szakaszokat, húrtrapézokat kapunk. Az ABED húrtrapézból : AB = ED. CEAB húrtrapézból : CB = AE. DABC húrtrapézból : AB = CD Eddig : AB = ED = CD EBCD húrtrapézból : CB = ED Tehát : AB = ED = CD = CB = AE Tehát ez egy olyan ötszög, amelynek minden oldala egyenlő, vagyis szabályos. Hol van a hiba a logikámban?

[411] HoA2006-04-15 15:52:08

[409] után a 68. - 70. - 71. feladatok tkp. a [388] -ban felvetett egyenlőtlenség szemléletes bizonyításai. Adjuk fel 72. feladat-ként az egyenlőtlenség trigonometriai - ábrát nem igénylő - bizonyítását.

Érdekességképpen megemlítem, hogy a tételre a Matematikai Versenytételek 1897(!!)-i egyik feladataként is rátaláltam. Igaz, ott az állítás nem ilyen éles, csak a

sin(\alpha/2) * sin(\beta/2) * sin(\gamma/2) \le 1/4

igazolását tűzték ki. A megoldás 1/8 -ra is szerepel.

Előzmény: [409] BohnerGéza, 2006-04-12 00:44:01
[410] Hajba Károly2006-04-12 07:38:44

Kedves Géza!

Ha jól vettem az adást, akkor gyakorlatilag a szögek és térszögek közötti összefüggést, hasonlóságot az érintő ill. érintősík elfordulás, elmozdulás mértékében kell, lehet keresni.

Mégegyszer köszi a tájékoztatód.

Előzmény: [405] Kós Géza, 2006-04-05 13:21:05
[409] BohnerGéza2006-04-12 00:44:01
[408] BohnerGéza2006-04-12 00:42:11

A következő hozzászólásban használom az alábbiakat:

[407] Vonka Vilmos Úr2006-04-07 11:25:57

Legyenek az ellipszis adott pontjai A, B, C, D, E!

1. A Pascal-tétel segítségével szerkeszthető pl. a C és D pontbeli érintő, ezek metszéspontját jelölje F!

2. Mivel F polárisa a CD egyenes, a CD irányához konjugált irányú átmérő áthalad az F ponton, valamint CD felezőpontján (G) is. Tehát FG (e) az ellipszis egy átmérő egyenese.

3. Legyen a D-n keresztül e-vel húzott párhuzamos egyenes f! A Pascal-tétel segítségével szerkeszthető f és az ellipszis másik metszéspontja (I). Az e-hez konjugált átmérő áthalad DI felezőpontján (J) és párhuzamos CD-vel: legyen ez a g egyenes.

4. A Steiner-szerkesztés segítségével megszerkeszthetőek e és g metszéspontjai az ellipszissel (LM, NO). Így megkaptuk az ellipszis egy konjugált átmérőpárját.

5. Egy konjugált átmérőpár ismeretében pl. Rytz-szerkesztéssel szerkeszthetőek az ellipszis tengelyei.

Előzmény: [406] Morci, 2006-04-06 21:59:18
[406] Morci2006-04-06 21:59:18

Üdv!

Segítséget szeretnék kérni. Netes keresőben itt dobott ki Pascal tétellel kapcsolatban infót, így gondoltam itt próbálok segítséget kérni.

Az a problémám, hogy egy ellipszis pontjai adottak, s nekem meg kell szekesztenem az ellipszist. A Pascal tétel alapján pontszámot tudok "bővíteni", illetve az itt talált ábra alapján még a nagytengely irányát is meg tudom határozni, viszont az ellipszis és a kör affin aránya ismeretlen, ezért nem lehet ezzel tovább lépni az ellipszis felől...

Van valamilyen módszer ellipszis szerkesztésére ha pár alkotó pontja adott csak?

Előre is köszönöm a segítséget!

[404] Hajba Károly2006-04-06 08:30:41

Kedves Géza!

Tájékoztatód köszönöm, elkezdem megemészteni. :o)

Előzmény: [405] Kós Géza, 2006-04-05 13:21:05
[405] Kós Géza2006-04-05 13:21:05

Kedves Károly,

Már régóta tervezgetem, hogy egyszer majd kimerítőbben válaszolok a kérdésedre, de eddig nem jutottam hozzá. Lehet, hogy most is csak egy részét írom le.

A térben egy csomó dolog nem igaz úgy, mint a síkon, egyes dolgok pedig teljesen elvesznek. Pl. a síkon egy üres rácsháromszög, aminem a csúcsai rácspontok, de sem a kerületén, sem a belsejében nincsenek további rácspontok, mindig 1/2 területű. A térben a megfelelő állítás nem igaz, egy üres rácstetraéder térfogata akármilyen nagy lehet.

A szögek összegének létezik a térben megfelelője, de nincs szoros kapcsolata a csúcsoknál előforduló térszögek összegével. Ha például egy tetraédert kilapítasz úgy, hogy konvex négyszöggé fajul, a térszögek mindegyike 0-hoz fog tartani.

Ami a szögek összegének megfelel, az a görbület integrálja. A síkban egy egyszerű zárt töröttvonal esetén a külső szögek összege 2\pi, illetve egy kétszer folytonosan differenciálható egyszerű görbén a görbület ívhossz szerinti integrálja 2\pi. Ha a görbe/töröttvonal nem egyszerű, akkor az eredmény 2\pi-szer az irány körülfordulásainak száma.

A térben egy elég sima, egyszerű zárt felületen a Gauss-görbület felszín szerinti integrálja mindig 4\pi.

A poliédereket tekinthetjük sima felületek limeszének. pl. Minden élt lekerekítünk egy hengerpalásttal, a csúcsokat pedig gömbfelületekkel. (Most tekintsünk el attól, hogy ez nem mindig lehetséges, mondjuk szorítkozzunk a konvex poliéderekre.) A síklapokon és a hengerpalástokon a Gauss-görbület 0. A csúcsoknál a görbület integrálja a megfelelő gömbsokszög területe. A gömbsokszögeket összetolhatjuk egyetlen gömbbé, aminek a felszíne 4\pi.

A gömbsokszögecskék területét másképp is kiszámolhatjuk.

Számoljuk össze egy csúcsnál a lapok szögeit. (Nem a lapsíkok közötti szögeket, hanem mindegyik lapnak az adott csúcsnál levő szögét.) Ha a szögek \alpha1,...,\alphan, akkor a gömbsokszög szögei \pi-\alpha1,...,\pi-\alphan, a területe pedig 2\pi-(\alpha1+...+\alphan). Ha ezt az összes csúcsra kiszámoljuk és összeadjuk, akkor a csúcsok számát meg kell szoroznunk 2\pi-vel és ki kell vonnunk az összes lap összes szögének összegét.

Legyen a csúcsok, élek, lapok száma C, E és L. Az egyes lapok éleinek száma legyen e1,...,eL. Ekkor persze e1+...+eL=2E, mert minden él két laphoz tartozik. Az i-edik lapon a szögek összege (ei-2)\pi, az összes lap összes szögének összege tehát (e1-2)\pi+...+(eL-2)\pi=(e1+...+eL-2L)\pi=(E-L)2\pi.

A görbület integrálja a teljes felületen C.2\pi-(E-L)2\pi=(C-E+L)2\pi=4\pi.

A dolog egyrészt vicces, mert megjelenik a poliédertétel (C-E+L=2), ugyanakkor mindezt Gauss-görbület integrálása nélkül is tudtuk, és sem lap, sem térszögek nem jelentek meg...

Előzmény: [402] Hajba Károly, 2006-04-03 23:35:54
[403] HoA2006-04-04 13:56:15

A [388] egyenlőtlenség és a 68. feladat közötti kapcsolatról:

Legyen BOC \angle=\alpha , COD \angle=\beta , DOA \angle=\gamma,\alpha+\beta+\gamma=\pi . Ekkor OAF \angle=\alpha/2 , DO = R ; FO = m = R*sin(\alpha/2), FA = d = R*cos(\alpha/2), FOD \angle=\epsilon=\beta-(\pi/2-\alpha/2)=\beta+\alpha/2-\pi/2

FD2=r2=DO2+FO2-2*DO*FO*cos\epsilon=R2*(1+sin2(\alpha/2)-2*sin(\alpha/2)*cos\epsilon)

n2=d2-r2=R2*(cos2(\alpha/2)-1-sin2(\alpha/2)+2*sin(\alpha/2)*cos\epsilon)=R2*(-2*sin2(\alpha/2)+2*sin(\alpha/2)*cos\epsilon)=

2R2*sin(\alpha/2)*(cos\epsilon-sin(\alpha/2)) .

A [388] egyenlőtlenségben válasszuk úgy a betűzést, hogy \beta\ge\gamma teljesüljön és ennek megfelelően legyen D közelebb A-hoz mint C-hez. Ekkor \epsilon=\beta+\alpha/2-\pi/2\ge\beta/2+\gamma/2+\alpha/2-\pi/2=0 , cos\epsilon=sin(\beta+\alpha/2) , n2=2R2*sin(\alpha/2)*(sin(\beta+\alpha/2)-sin(\alpha/2))

Felhasználva a sin(u+v) - sin(u-v) = 2*cos(u)*sin(v) azonosságot

n2=4R2*sin(\alpha/2)*cos((\beta+\alpha)/2)*sin(\beta/2)=4R2*sin(\alpha/2)*sin(\gamma/2)*sin(\beta/2)

Előzmény: [388] BohnerGéza, 2006-03-14 12:02:36
[402] Hajba Károly2006-04-03 23:35:54

Üdv!

A Wolfram-on kicsit kutakodva ill. az SH Atlaszban találtam még néhány dolgot, de egy kicsit elgondolkoztam én is a témán. (Magad uram, ha szolgád nincsen.:o)

Ismert, hogy egy gömbháromszög területe T_\Delta = \sum_{i=1}^3\alpha_i-\pi. Ezen minimális gömbi sokszöghöz adott szomszédos pontpárjukat egyesítve újabb háromszög illeszthető. Így tetszőleges n-sokszög állítható elő, akár konkáv is. Ezen sokszög területe, melyet nem nehéz belátni, T=\sum_{i=1}^n\alpha_n-(n-2)*\pi

Ezen gömb középpontjából kiinduló és a sokszög pontjaira illesztett félegyenesek a pontok sorrendje szerint a félegyenesek közötti síkok által meghatározott térrész a T-vel arányos térszöget határoz meg.

A gömbi sokszög adott csúcspontjára illesztett és a gömböt érintő sík a csúcsba befutó két gömbi egyenes és gömbközéppont által meghatározott két síklapra merőleges. Ezért a két síklap által bezárt szög azonos a csúcsponti szöggel.

Fentiekből következik, hogy egy síkidom adott csúcspontjához tartozó szomszédos lapok által meghatározott szögek összege és a lapok által meghatározott térszög mértéke között szoros összefüggés áll fenn a fenti képlet szerint.

Újabb érdekes összefüggések adódnak egy idom lapszögeinek és térszögeinek összegei között, de erről később, ha addig valaki nem tesz be egy ezirányú összefoglaló linket. Mert biztos van erről irodalom, legfeljebb még nem bukkantam rá.

Előzmény: [398] Lóczi Lajos, 2006-03-29 22:21:15
[401] HoA2006-04-03 15:36:57

1.) C-t - és ezzel BC-t - rögzítve, a CD * DA szorzat akkor maximális, amikor az ACD \Delta területe, hiszen \delta és így sin\delta is állandó, vagyis ha D az AC ív felezőpontja (ld. Jenei [395]). Ugyanez mondható D rögzítése esetén a BC * CD szorzatról, tehát C a BD ív felezőpontja. A két feltétel együttesen akkor következik be, ha C és D helyzete a [386] második ábrája szerinti.

2.)Így már világos a kapcsolat a [388]-ban feladott egyenlőtlenséggel: a vizsgált szorzat tkp.

2R*sin\frac{\alpha}2 * 2R*sin\frac{\beta}2 * 2R*sin\frac{\gamma}2

, ahol \alpha+\beta+\gamma=\pi . Azon még gondolkodom, a 68-as feladat hogyan következik az egyenlőtlenségből.

Előzmény: [399] BohnerGéza, 2006-03-30 08:02:53
[400] Hajba Károly2006-03-30 14:21:40

Köszi. Amit nyelvtudás nélkül ki lehet belőle hámozni, azt eddig is sejtettem.

Gyakorlatilag az érdekelne, hogy a síklapú testek csúcsaihoz tartozó térszögek összege milyen szabályosságot mutat. Mennyire analóg ill. milyen módon analóg a 2D-s szabályokkal. \sum_{i=1}^n\alpha_i=(n-2)*\pi Egy-egy csúcshoz tartozó síkszögek és térszög között milyen összefüggések állnak fenn.

Azt gyanítom, hogy azonos csúcsszámú testekhez nem feltétlenül azonos nagyságú térszög összeg tartozik. Talán a csúcshoz tartozó síkszögek valamiféle összesítése állandó, vagy esetleg a síklapok által közrezárt szögek összege?

Előzmény: [398] Lóczi Lajos, 2006-03-29 22:21:15
[399] BohnerGéza2006-03-30 08:02:53

Köszönöm HoA-nak és Jenei Attilának a 68. ill. 70. feladatra adott megoldásait! Még eggyel bővítem a kapcsolódó feladatokat:

71. feladat: Az adott AB átmérőjű félkörön a 68. feladat ábrájának megfelelő sorrendben ([385] hozzászólás) helyezkedik el a C és D pont. Hol vannak ezek, ha a BC*CD*DA maximális?

[398] Lóczi Lajos2006-03-29 22:21:15

Csak eszembe jutott

http://mathworld.wolfram.com/SolidAngle.html

De innentől ez már inkább gömbi geometria.

Előzmény: [397] Hajba Károly, 2006-03-27 13:21:05
[397] Hajba Károly2006-03-27 13:21:05

Üdv!

Egy kis elméleti fejtágításra lenne szükségem a térszögek terén. Az alapfogalmak (térszög, szteradián) meghatározásán kivül nem sok mindent ad ki a Kugli.

Előre is köszönök minden segítséget ill. magyarnyelvű linket.

[396] HoA2006-03-24 14:34:56

Kedvet kapva Jenei Attilától egy cosinus tételes megoldásra, mégegyszer a 68. feladatról : Odáig, hogy k belülről érinti f-et ld. a 386-os hozzászólást. Legyen az f2 félkörív középpontja P. AD-t rögzítve n2=AF2-FD2=AD2-2*AD*FD*cos\delta=AD(AD-2*FD*cos\delta) . Mivel D az f2 köríven kívül van, \delta hegyesszög, a kifejezés akkor a legnagyobb, ha DT=FD*cos\delta , FD merőleges vetülete AD-re a legkisebb. Ez pedig akkor áll elő, ha FT az f2 körív AD-re merőleges érintője. De ekkor PF || AD, POF és AOD \Delta-ek hasonlóak, PF = PO -> AD = AO, vagyis AD az f félkörben sugár hosszúságú húr.

Előzmény: [395] jenei.attila, 2006-03-22 11:48:06
[395] jenei.attila2006-03-22 11:48:06

A szóbanforgó érintési pont legyen E.

BE2=BD2-DE2=BD2-DC2=AB2-AD2-DC2

Ezért BE akkor maximális, ha AD2+DC2 minimális.

AD2+DC2=AC2+2*AD*DC*cos \delta

. Mivel AC állandó és cos \delta állandó és negatív AD*DC-nek kell maximálisnak lenni. Vagyis mivel sin \delta állandó és pozitív az ADC háromszög területének kell maximálisnak lenni. Ez pedig akkor maximális, ha D az AC ív felezőpontja. Nem használtuk ki, hogy C az AB ív felezőpontja.

Előzmény: [389] BohnerGéza, 2006-03-17 07:55:20
[394] tothszivike2006-03-21 14:15:01

Köszönöm Sokat segítettél!!!

[393] HoA2006-03-21 09:55:50

A 70. feladat-ra van egy, a 68-ashoz hasonló trigonometriai megoldásom, ha nem lesz jobb, beírom. De szívesen látnék erre is meg a 68-asra is egy nem trigonometriait.

Előzmény: [389] BohnerGéza, 2006-03-17 07:55:20
[392] HoA2006-03-21 09:50:54

Ha arra gondolsz, hogyan kell elforgatni egy négyzetet a síkjában fekvő adott pont körül adott szöggel - és ekkor mindegy, a pont a négyzeten belül vagy kívül van - egy megoldás az ábra szerinti: az adott pontból a négyzet csúcsaiba húzott szakaszok mindegyikét elforgatjuk az adott szöggel. Az új végpontok az elforgatott négyzet csúcsai.

Előzmény: [391] tothszivike, 2006-03-20 19:00:39
[391] tothszivike2006-03-20 19:00:39

Segítséget szeretnék kérni! Hogyan kell elforgatni egy négyzetet egy azon belüli ponton keresztül mert nekük az osztályban mindenkinek trapéz jött ki a tanárnő meg nem hajlandó megmutatni!

[389] BohnerGéza2006-03-17 07:55:20

A 384. hozzászólásban HoA által felvetett kérdés miatt alakult ki a következő feladat:

70. feladat: Az AB átmérőjű félkörív felezőpontja C. D az AC íven mozoghat. Mikor lesz leghosszabb a B-ből a D középpontú, C-n átmenő körhöz húzott érintőszakasz?

[388] BohnerGéza2006-03-14 12:02:36
[387] axbx2006-03-13 20:48:33

Nem megy nékem az geometria..

[386] HoA2006-03-10 16:17:31

Köszönöm az ábrát.

Legyen az AB átmérőjű f félkör középpontja O, sugara R. F rajta van f A-ból vett 1/2 arányú kicsinyítésén vagyis az AO átmérőjű f2 félkörön. n hossza A pont k-ra vonatkozó hatványának négyzetgyöke. Az AF távolságot d-vel, k sugarát r-rel jelölve n2=(d+r)(d-r)=d2-r2 . C-t - és ezzel F-et és d-t - rögzítve ez akkor a legnagyobb amikor r a legkisebb, vagyis amikor k belülről érinti f-et. Ekkor D az OF egyenes és f metszéspontja, jelöljük Dm-mel.

C-t az f félkörön mozgatva, C és F helyzetét a 0 és \frac{\pi}2 közé eső BAC = OAF = \alpha szöggel jellemezve, mivel F rajta van OA Thalesz-körén

d=Rcos\alpha,r=R(1-sin\alpha)

n2=R2(cos2\alpha-(1-sin\alpha)2)=R2(cos2\alpha-1-sin2\alpha+2sin\alpha)=R2(cos2\alpha-cos2\alpha-sin2\alpha-sin2\alpha+2sin\alpha)=

=2R2(sin\alpha-sin2\alpha)=2R2(sin\alpha)(1-sin\alpha)

Ez pedig a számtani és mértani közép egyenlőtlenség miatt akkor a legnagyobb, ha sin\alpha=1-sin\alpha=1/2 Ez a vizsgált tartományban \alpha=30o -nál következik be. Így DOA szög = CBA szög = 60o , A, D, C és B pontok egy szabályos hatszög egymás utáni csúcsai. sin\alpha=1/2 -t helyettessítve  n^2 = R^2 /2 , n = R / \sqrt2

Előzmény: [385] BohnerGéza, 2006-03-09 20:26:52
[385] BohnerGéza2006-03-09 20:26:52

A 68-as feladat ábrája.

[384] HoA2006-03-09 17:53:28

- Az én értelmezésemben a rögzített AB átmérőjű félköríven C és D csak ACDB vagy ADCB sorrendben lehetnek.

- F az AC szakasz vagy az AC ív felezőpontja?

Előzmény: [382] BohnerGéza, 2006-03-09 12:15:40
[383] BohnerGéza2006-03-09 14:09:04
[382] BohnerGéza2006-03-09 12:15:40

68. feladat: C és D a rögzített AB átmérőjű félköríven vannak ABCD sorrendben. Legyen F az AC felezőpontja, k az F középpontú, D-n átmenő kör és n az A-ból k-ig húzott érintőszakasz. Hol van C és D, ha n a lehető leghosszabb? Mekkora ekkor n? (AB-hez képest.)

[381] hobbymatekos2006-03-08 14:48:24

Sziasztok. Én úgy gondolom: a szabályos háromszög oldallapok súlypontjaiba a lapok területével azonos skalárok (mint tömeg pontrendszer) tömegközéppontjában lesz a tetraéder súlypontja. (Vagyis a statikai nyomatékok vektorainak bármely geometriai pontra számitott eredő nyomaték vektorának a súlypontba redukáltja nullvektor.)

Előzmény: [378] BohnerGéza, 2006-02-23 23:30:05
[379] axbx2006-02-27 16:34:28

Már a véleményemet se mondhatom el.. (Fődmívelö) Azért nem vót szép, hogy bannoltak.

Na csak Az lenne a kérdésem, hogy hogyan kell bizonyítani a kör egyenletét?

[378] BohnerGéza2006-02-23 23:30:05

Azt hiszem, az előző feladat után természetesen adódik, az érdeklődés miatt is, ha a térbeli analóg feladatot is kitűzöm.

67. feladat: Hol van a homogén vékony lemezből álló ABCD tetraéderhéj fizikai értelemben vett súlypontja.

[377] jonas2006-02-23 23:13:42

Tényleg.

Előzmény: [376] Káli gúla, 2006-02-23 23:10:28
[376] Káli gúla2006-02-23 23:10:28

De, ott van. The Spieker circle ... , X(10), is the centroid of the perimeter of ABC.

Előzmény: [375] jonas, 2006-02-23 22:57:55
[375] jonas2006-02-23 22:57:55

A katalógus szerint ennek a neve X(10) = SPIEKER CENTER. Érdekes módon ez nem említi, hogy ez lenne a homogén keretű háromszög súlypontja. Vagy én néztem volna el valamit?

Előzmény: [372] jonas, 2006-02-23 22:45:46
[374] Hajba Károly2006-02-23 22:55:43

Lusta vagyok feliratozni, de ez jonas magyarázatának melléklete. :o)

Előzmény: [372] jonas, 2006-02-23 22:45:46
[373] Hajba Károly2006-02-23 22:47:59

Huú! De nagy lett itt a forgalom. :o)

És ez a pont könnyen szerkeszthető. Az oldalfelezőből a szemközti csúcs szögfelezőjével párhuzamost húzunk.

Előzmény: [369] jonas, 2006-02-23 22:34:34
[372] jonas2006-02-23 22:45:46

Akkor a keresett pontot úgy is megkaphatjuk, hogy az eredeti háromszög beírt körének középpontját -1/2-szeresére nagyítjuk az eredeti háromszög súlypontjából, vagy úgy is, hogy az eredeti háromszög csúcsait b+c,c+a,a+b arányban súlyozzuk. (Hol is van az a háromszög-nevezetes-pont-katalógus?)

Előzmény: [369] jonas, 2006-02-23 22:34:34
[371] lorantfy2006-02-23 22:41:25

Az FcEFa háromszög hasonló CDB háromszöghöz, így az Fc-ból induló szögfelező a szemközti oldalt éppen (b+c)/a arányban osztja. Tehát S pont az FaFbFc háromszög szögfelezőinek metszéspontja, vagyis a beírt körének középpontja.

Előzmény: [369] jonas, 2006-02-23 22:34:34
[370] lorantfy2006-02-23 22:35:20

Igen! A szerkesztés ábrájából is látszik és könnyen bizonyitható.

Előzmény: [368] jonas, 2006-02-23 22:29:56
[369] jonas2006-02-23 22:34:34

Azzal hogy az egyes oldalakat drótból készítenénk el, egyenértékű, ha az oldalak felezőpontjába rakunk az oldalhosszal arányos súlyokat, és ezeknek a súlypontját keressük. Na de a középvonal-háromszög oldalai fele olyan hosszúak, mint az eredeti háromszög megfelelő oldalai. Ezért a középvonal-háromszög csúcsait kell súlyozni a szemközti oldalakkal, így pedig a beírt körét kapjuk.

Előzmény: [368] jonas, 2006-02-23 22:29:56
[368] jonas2006-02-23 22:29:56

Megvan. A középvonal-háromszög beírt körének középpontja lesz, mindjárt elmondom, miért.

Előzmény: [367] jonas, 2006-02-23 22:26:52
[367] jonas2006-02-23 22:26:52

Nézzük csak. Nevezetes pont. Egy nagyon hosszú egyenlőszárú háromszögnek a felénél van, tehát nem lehet a magasságpont, a beírt kör középpontja, a súlypont vagy a Feuerbach kör középpontja, csak a körülírt kör középpontja lehetne. Az viszont nyilván nem lehet, mert az a háromszögön kívül is lehet, a drót súlypontja viszont nem. Akkor kevésbé nevezetes pont lesz.

Előzmény: [362] BohnerGéza, 2006-02-23 08:15:11
[366] lorantfy2006-02-23 22:22:21

Szerkesszük meg ezt a speciális súlypontot!

Az oldalfelező pontokba helyezzünk az oldalak hosszának megfelelő tömegpontokat.

A 'bc' szakaszt c/b arányban kell osztanunk. A szögfelező b/c arányban osztja. Tükrözzük ezt a pontot a szakasz felezőpontjára és helyezzünk ebbe a pontba b+c tömegpontot.

Már csak az 'a b+c' szakaszt kell (b+c)/a arányban felosztani. A C-ből induló szögfelező BD szakaszt a/(b+c). Ehhez hasonló kisháromszöget alakítunk ki és ebben a szögfelező azonos arányban osztja a szemközti oldalt.

Már csak tükröznünk kell ezt a pontot a felezőpontra. Ez lesz az S súlypont. Remélem idáig jó!

Előzmény: [362] BohnerGéza, 2006-02-23 08:15:11
[365] Hajba Károly2006-02-23 21:08:51

Jogos. :o)

Drótnak drót, csak nem homogén az én elképzelésem. Egymáshoz viszonyított fajlagos súlyuk arányos az S pont és adott oldal közötti távolsággal.

Visszavonva.

Előzmény: [364] Káli gúla, 2006-02-23 19:19:48
[364] Káli gúla2006-02-23 19:19:48

A maradék általában nem egyforma széles a három oldalnál.

Előzmény: [363] Hajba Károly, 2006-02-23 18:37:13
[363] Hajba Károly2006-02-23 18:37:13

66. Tekintsük a teljes ABC\Delta-t és képezzünk egy nála kisebb hasonló \Delta-t úgy, hogy a súlypontjuk egybeessen és az egymásnak megfelelő oldalak rendre párhuzamosak legyenek. A két \Delta közös súlypontja is az S pontban lesz, így elvileg az eredeti \Delta-ünkből elvesszük a kisebb \Delta-t, akkor ennek a "lyukas" \Delta-nek is S marad a súlypontja. Ha növeljük a lyukat, akkor szélső esetben a feladatbali drótvázhoz jutunk.

Így az ABC\Delta-nek is S a súlypontja.

Előzmény: [362] BohnerGéza, 2006-02-23 08:15:11
[362] BohnerGéza2006-02-23 08:15:11

66. feladat: Hol van egy homogén, vékony drótból készült ABC háromszögnek a fizikai értelemben vett súlypontja? ( ABC-hez kapcsolható nevezetes pont lesz.)

[361] nadorp2006-02-19 08:52:11

Köszönöm a megoldást. Először én is hasonlóan csináltam, aztán rájöttem, ez a megoldás is a háttérben tulajdonképpen - ahogy Te is írtad az elején - trigonometrikus ( az x<tg x összefüggést használja ), pedig nincs is rá szükség. Legyen a sokszög oldala egységnyi és a beírt kör sugara r. Ekkor K=n, T=\frac{nr}2. Ebből \frac{K^2}T=\frac{2n}r. A beírt kör területe kisebb a sokszög területénél, azaz r^2\pi<\frac{nr}2. Ebböl következik, hogy \frac{2n}r>4\pi

Előzmény: [360] qer, 2006-02-17 18:55:53
[360] qer2006-02-17 18:55:53

Megoldás a 65. feladatra: leírva nincs szögfüggvény, de végülis ott van...

Legyen AB az n-szög egyik oldala, a kör sugara egységnyi. Ekkor AOB szög \frac{2\pi}{n}, AB-t jelöljük a-val, OF-t m-mel.

Nyílván a=\frac K n, T=n \frac {am} 2. Az \frac {a^2} {\frac {am} 2} hányadost átalakíthatjuk a következő alakra: \frac 1 n \frac{K^2} {T}. Tehát a bizonyítandó egyenlőtlenséget így is írhatjuk: n\frac {a^2} {\frac {am} 2} > 4\pi.

Kis átalakítások után a következő alakra hozható az egyenlőtlenség: \frac {\frac{a}{2}} {m} > \frac \pi n.

\frac \pi n az AOF (vagy A'OF') szöggel egyenlő. Mivel \frac {A'F'} {OF'} = \frac {AF} {OF}, ebből következik (OF'=1), hogy A'F' = \frac {\frac{a}{2}} {m}.

Mivel n>2 egész szám ezért, az AOF szög kisebb mint derékszög. Ezért már csak azt kell bebizonyítani, hogy AF' ív kisebb mint A'F'. Mivel AOF' körcikk benne van az A'OF' háromszögben, ezért a területe is kisebb. A körcikk területe \frac 1 2 AF' (az AF' ívmérték), a háromszögé pedig \frac 1 2 A'F', azaz igaz az A'F' > AF' egyenlőtlenség. Mivel az átalakítások ekvivalensek voltak ezért igaz a kiinduló egyenlőtlenség is.

Előzmény: [349] nadorp, 2006-02-06 17:30:46
[359] BohnerGéza2006-02-16 00:11:44

A 64. feladat megoldásához: Vegyük fel például az AP=f szakaszt, az A kp-ú c sugarú kör vonal a B pont számára ( vB ) és a b sugarú pedig a C számára ( vC ). A szögfelező tételtből tudjuk: ha a vC-re alkalmazzuk a P kp-ú lambda = -c/b arányú hasonlóságot, B számára kapunk egy második vonalat (vC'). ( A szerkesztéshez fölhasználunk egy P-től b-re lévő S segédpontot és képét ( S') P-től c-re, SPS' sorrendben. A vC' A' kp-ú c sugarú kör lesz. )

Megjegyzés: Például az AB szakaszból kiindulva B kp-ú lambda=(b+c)/c hasonlósággal is megoldható a feladat P képe lesz C.

[357] Hajba Károly2006-02-12 18:58:52

Részemről jöhet az arányos szerkesztés nélküli megoldás. Talán a szögfelezők "tétele" C2 pontjának megszerkesztése a megolodás, de arra még nem jöttem rá, hogy tudom az ismert adatokból megszerkeszteni.

Előzmény: [354] lorantfy, 2006-02-11 20:51:42
[356] lorantfy2006-02-11 21:31:49

Kedves HoA!

Mindkét megjegyzésed helyénvaló. Köszönet érte! A második még egyszerűbbé teszi a megoldást.

Előzmény: [355] HoA, 2006-02-11 20:56:28
[355] HoA2006-02-11 20:56:28

Kedves László!

Megoldásod szép és egyszerű. Két megjegyzés:

- az utolsó előtti sorban "A c oldalt felosztjuk a:c arányban" helyett "A c oldalt felosztjuk b:c arányban" a helyes, és így az ábra legalsó "a" betűje helyett is "b" a jó.

- Az AQP \Delta egyenlőszárú volta "ránézésre" is következik abból, hogy két szöge egyenlő, mert f szögfelező , PAC és APQ váltószögek

Előzmény: [352] lorantfy, 2006-02-11 13:12:31
[354] lorantfy2006-02-11 20:51:42

Köszönöm! Így már világos Nagyon ügyes.

Már csak egy arányos szerkesztés nélküli megoldást várunk!

Előzmény: [353] Hajba Károly, 2006-02-11 19:49:37
[353] Hajba Károly2006-02-11 19:49:37

Kedves László!

Az arányosításnál egy kis magyarázatbeli bakit elkövettem, de a szerkesstés menete szabályos. Felszerkesztettem a Te ábrád szerint újra:

t illeszkedik B-re és merőleges f-re. P tükörképe t-re P'. Így az ACP\Delta ill. ABP'\Delta hasonlók. AP'\tof.'.

\frac{b}{f}=\frac{c}{f'} \to \frac{f'}{f}=\frac{c}{b}.

S a szerkesztés menete:

Felveszem az f egyenest és felmérem rá AP szakaszt. Elkészítem a \frac{c+b}{2} : b arányosítást f-re. Ez P". Erre a pontra merőlegesen felszerkesztem a t egyenest, majd A-ból rámérem c-t, ez adja a B pontot. Innen már megegyezik a tieddel és nyilvánvaló.

Előzmény: [352] lorantfy, 2006-02-11 13:12:31
[352] lorantfy2006-02-11 13:12:31

Kedves Károly!

Lehet, hogy jó a megoldásod, de én nem értem a magyarázatot. Fölteszem az enyémet, ez is arányos szerkesztéssel megy.

64. feladat megoldása: Adott az ABC háromszögben a b, c oldal és az f szögfelező. A szögfelező a BC oldalt P pontban metszi. Legyen CP=p és PB=q. A szögfelező tételből következik, hogy \frac{p}{q} =\frac{b}{c}.

Húzzunk párhuzamost a P pontból AC-vel, ez az AB oldalt Q pontban metszi. Legyen PQ=x, AQ=y és QB=z.

Ekkor a párhuzamos szelők tételéből: \frac{y}{z}=\frac{p}{q}=\frac{b}{c}. Vagyis y=z\frac{b}{c}

Másrészt a közbenső szakaszokra, QBP és ABC háromszögek hasonlóságából: \frac{x}{z}=\frac{b}{c}. Vagyis x=z\frac{b}{c}

Tehát x=y. Az AQP \Delta egyenlő szárú. A szerkesztés innen már egyszerű. A c oldalt felosztjuk a:c arányban. Q pontból AQ-val, A-ból f-el körözve kapjuk a P pontot. Majd A-ból b-vel körözve BP egyenesből kimetsszük C-t.

Előzmény: [351] Hajba Károly, 2006-02-07 13:12:51
[351] Hajba Károly2006-02-07 13:12:51

Legyen az ABC\Delta a megoldás háromszöge! Ismert az a, b, f szakasz.

Egy kis elméleti megfontolás a megoldás szerkesztéséhez: Tükrözzük a B és F pontokat az f-re merőleges és A-n átmneő t tükörtengelyre! Ekkor a \frac{F'A}{AC}=\frac{FB}{BC} ill. \frac{a+b}{2b} arányosítással megszerkeszthető a \frac{CF+CF'}{2} szakasz. Innen már nem nehéz kitalálni a szerkesztés menetét.

Előzmény: [346] lorantfy, 2006-02-05 13:46:45
[350] nadorp2006-02-06 17:32:00

Természetesen 65. feladat

[349] nadorp2006-02-06 17:30:46

64.feladat. Egy szabályos n-szög területe T, kerülete K. Bizonyítsuk be szögfüggvények nélkül, hogy \frac{K^2}T>4\pi

[348] nadorp2006-02-06 17:17:45

Negyedik arányos szerkesztés alkalmazása nélküli ( tehát valamilyen direkt ) megoldás engem is érdekelne.

Előzmény: [346] lorantfy, 2006-02-05 13:46:45
[347] Lóczi Lajos2006-02-06 09:51:43

[Micsoda egybeesés. Épp tegnap olvastam a gyűjteményben valami okból ezt a feladatot :) ]

Előzmény: [346] lorantfy, 2006-02-05 13:46:45
[346] lorantfy2006-02-05 13:46:45

64. feladat: Szerkesszünk háromszöget, ha adott az egyik csúcsából induló két oldal és a szögfelező hossza ( a szögfelező egyenesének a háromszögbe eső szakasza)!

[345] Karácsony2006-01-18 10:05:49

köszönöm!!! a dolgozat egész jól sikerűlt, és sikerűlt elkapnom a fonalat!! egyébként nincs bajom a matekkal, de a legegyszerűbb dolgok szoktak a legtöbb fejtörést okozni. örülök, hogy vannak még ilyen rendes fiúk, akik segítenek a bajbajutottakon!! mégegyszer köszi: Dorka

Előzmény: [344] Sirpi, 2006-01-17 14:26:59
[344] Sirpi2006-01-17 14:26:59

A 2x-3y=7 attól egy egyenes egyenlete, hogy azok az (x;y) párok, melyekre teljesül ez az egyenlet, éppen egy egyenesen vannak. És ha erről az egyenesről kellenek pontok, könnyen tudunk generálni akárhányat. Pl. ha x=2, akkor ezt az egyenletbe beírva 4-3y=7, ahonnan y=-1, vagyis rögtön kaptuk, hogy a (2;-1) pont rajta van az egyenesen és ezzel a módszerrel újabb pontok is generálhatók.

***

Másik irány: például az A(1;4) és a B(-3;-2) pontokon áthaladó egyenest keressük.

Ahhoz, hogy egy egyenes átmenjen A-n, y=a(x-1)+4 alakúnak kell lennie valamilyen a paraméterrel, hiszen ekkor A koordinátáit behelyettesítve a bal oldal 4, a jobb pedig a(1-1)+4=4 a értékétől függetlenül. a pedig megkapható abból, hogy az egyenes átmegy B-n: -2=a(-3-1)+4, azaz -6=(-4)a, vagyis a=3/2, így az egyenes egyenlete: y=3/2(x-1)+4. Kettővel szorozva: 2y=3x-3+8, ezt pedig rendezve kapjuk, hogy 3x-2y=-5, könnyen utána lehet számolni. Remélem, ez segített valamennyit.

Előzmény: [343] Karácsony, 2006-01-17 13:33:38
[343] Karácsony2006-01-17 13:33:38

előszőr is köszönöm és hálám örökké üldözni fog. igazából az a baj, hogy elég rossz matektanárunk van és mindig már az elején elvesztem a fonalat! a kérdés az, hogy: 2x-3y=7 egyenesnek adjam meg a két különböző pontját és számítsam ki ezek pontok távolságát. a távolságot már ki tudom számolni, de a pontokig nem jutok el. és azt sem tudom, hogyha két pont koordinátája van meg, akkor abból hogy lesy egzenlet!!

Előzmény: [342] Sirpi, 2006-01-17 13:03:30
[342] Sirpi2006-01-17 13:03:30

Egy vektor hossza a koordinátái négyzetösszegének négyzetgyöke, azaz:

a(-2;7), b(10;2) esetén a-b=(-2-10;7-2)=(-12;5)

Ennek hossza \sqrt{(-12)^2+5^2}=13

c(3;1) esetén 3c=(9;3)

Ennek hossza: \sqrt{9^2+3^2}=\sqrt{90}=3\sqrt{10}, nem meglepő módon épp háromszor akkora, mint c hossza.

***

Az egyenes egyenleténél pedig hogy érted, hogy hogy lesz? Pl. 2 ponton át akarsz egyenest húzni, és annak érdekel az egyenlete? Vagy egy ponton át adott vektorral párhuzamos egyenes egyenlete érdekel? Csak mert ezeket mind meg lehet mondani, a fő kérdés az, hogy Téged mi érdekel pontosan.

Előzmény: [341] Karácsony, 2006-01-17 12:17:57
[341] Karácsony2006-01-17 12:17:57

KOOrdináta geometria a halálom! szeretnék még segítséget!!! a vektor(-2;7) b vektor (10;2) a kérdés a-b vektor hossza???

a másik: a vektor (3;1) kérdés 3a vektor hossza???

ja és még azt nem értem, hogy hogy lesz az egyenes egyenlete!!!

köszike: Dorka ui: bocsika, de meg fogok bukni, mert csak ebből irat a tanár!!

[340] Karácsony2006-01-17 12:11:06

köszönöm szépen!!!!!

Előzmény: [339] Sirpi, 2006-01-17 10:28:48
[339] Sirpi2006-01-17 10:28:48

Fogd fel úgy, mint kétismeretlenes lineáris egyenletrendszert. Törttelenítve:

e:  3x-2y=12

f:  3x-y=6

Másodikból kivonva az elsőt kapjuk, hogy y=-6, ezt pedig bármelyikbe behelyettesítve kijön, hogy x=0.

Tehát a metszéspont a (0;-6) pont.

Előzmény: [338] Karácsony, 2006-01-17 09:44:29
[338] Karácsony2006-01-17 09:44:29

lenne egy kérdésem, remélem tudtok segíteni. a feladat úgy szól, hogy meg kell határoznbom az e és f egyenesek metszéspontját, ha e: 0,5x-(1/3)y=2 f: 3x-y=6 előre is köszönöm

[337] lorantfy2006-01-12 00:55:18

A Menelaosz tétel és megfordításának biz. vektorokkal benne van Reiman István: Geometria és határterületei c. könyvében.

Előzmény: [336] philip, 2006-01-11 18:43:04
[336] philip2006-01-11 18:43:04

Nagyon megköszönném,ha valaki tudna nekem segíteni Menelaosz-tételének és annak megfordításának bizonyításában.Vagy az érintő négyszögekre vonatkozó Newton-tétel bizonyításában.

[335] 25012006-01-09 00:42:30

Bocs, hogy megint "belepofazok", de ha azt akarod megvizsgalni, hogy egy pont egy bizonyos koron belul van-e, akkor miert nem azt vizsgalod meg? Ehhez eleg az egyenlotlenseget tesztelni.

Azt gondoltam egyebkent, hogyha "masodfoku alakra" hozom neked az egyenletet, akkor majd "felismered jol", es alkalmazod a megoldokepletet. Tevedtem.

Előzmény: [330] Chryst, 2006-01-08 10:21:14
[334] Doom2006-01-08 13:06:24

"+u" a végén a "-u" helyett, egyébként stimmel:)

Előzmény: [333] jonas, 2006-01-08 12:08:01
[333] jonas2006-01-08 12:08:01

Igen, valóban kell egy abszolútérték bele.

Ha ez az egyenleted van, hogy

(x-u)2=r2-(y-v)2

akkor ebből így vonunk gyököt:


|x - u| = \sqrt{r^2 - (y - v)^2}

vagy kényelmesebben


x - u = \pm\sqrt{r^2 - (y - v)^2}

amiből


x = - u \pm\sqrt{r^2 - (y - v)^2}

Előzmény: [331] Chryst, 2006-01-08 10:59:05
[332] Chryst2006-01-08 11:12:05

Az a "-" a gyök előtt az ám "+" akar lenni, csak szerintem mellényúltam és nem vettem észre. Szóval így:

x=u+\sqrt{r^2-(y - v)^2}

Viszotn most valami nagyon elgondolkodtatott: Ha gyököt vontam, akkor oda nem kellene valahova egy abszolútérték?

Lehet, hogy hülyeség, de suliban is mindig elfelejtkeztem hasonló esetekben az abszolútértékekről, most meg (már) nem tudom, hogy kell-e vagy sem.

[331] Chryst2006-01-08 10:59:05

Jézusom!!!

Megvan a megoldás!!! És nagyon egyszerű. Csak nem értem, hogy eddig egyedül, magamtól hogy nem jöttem rá.

Leírom:

(x-u)2+(y-v)2=r2

rendezgetek egyet:

(x-u)2=r2-(y-v)2

gyököt vonok:

x-u=\sqrt{r^2-(y-v)^2}

megint rendezkedek, és már ki is derül milyen apróság nem jutott eddig eszembe:

x=u-\sqrt{r^2-(y-v)^2}

Hát igen, ismételten beigazolódott: "Az igazi zseniknek a legegyszerűbb dolgok jelentik a legnagyobb problémát" :)

A segítséget azért köszönöm

[330] Chryst2006-01-08 10:21:14

Köszönöm, de ilyesmiket én is kihoztam, csak mindegyikkel ugyanaz az egy baj volt:

x2-2xu+u2+(y-v)2-r2=0

Ebből ugyebár ezt kapom:

x2=2xu-u2-(y-v)2+r2 vagy x2-2xu=r2-u2-(y-v)2

Ez nekem azért rossz, mert ha én a programmal az x-et akarom kiszámoltatni, akkor az x csak egyszer szerepelhet az egyenletben, méghozzá x=? helyen.

Ezekben az egyenletekben szerintem első problémát az jelenti, hogy az egyik x előtt van egy u szorzó. Megpróbáltam eltenni máshová azt az u-t, de akkor meg az x2 alá került, valahogy így:

x=\frac{x^2+u^2+(y-v)^2-r^2}{2u}

és ez nekem már megint nem jó, mert onnan az x2-et még annyira sem tudom kivenni.

Lényegében nekem egy olyan egyenlet kell, mint amit akkor csinálunk, amikor több ismeretlenes egyenleteket számoltunk azzal a módszerrel, hogy az egyik ismeretlent kifejezem, amit meg kapok, azt behelyettesítem a másik egyenletbe. Amikor így fejezem ki az egyik ismeretlent, akkor nem szerepelhet a másik oldalon is az az ismeretlen, mert akkor nem tudnám kiszámolni a másik ismeretlent.

Szóval nekem egy ilyen egyenletre (vagy inkább képletre) lenne szükségem.

Nem akarok kört rajzolni, csak azt akarom megvizsgáltatni, hogy egy, a képernyőn elhelyezkedő pont egy bizonyos (előre megadott) körön belül van-e. Szóval szerintem mindenféleképpen ezzel az egyenlettel célszerű számolnom.

Tudom, hogy ezzel a képlettel még csak egy kör körvonalát kapom meg, de egyelőre még nem akartam egyenlőtlenséggel számolni, (ezzel: (x-u)2+(y-v)2<r2) mert ezzel ráérek majd akkor foglalkozni, amikor a progit írom. Ha egy kör körvonala megvan, az már fél siker, kiindulásnak elég lesz.

A további segítségeket előre is köszönöm

Előzmény: [329] 2501, 2006-01-08 02:59:52
[329] 25012006-01-08 02:59:52

(x-u)2+(y-v)2=r2

x2-2xu+u2+(y-v)2-r2=0

Innen mar remelem megy egyedul is. :)

(Lesz benne gyokvonas. Ha kort kell rajzolni, akkor vannak ennel sokkal hatekonyabb eljarasok.)

Előzmény: [328] Chryst, 2006-01-07 22:19:47
[328] Chryst2006-01-07 22:19:47

Programozok, és egy programban szeretnék kipróbálni valamit, amihez (koordináta geometriában) a kör egyenletéből ki kellene fejeznem az x-et, de egyedül nem tudok értelmes (aránylag egyszerűnek mondható) választ csinálni az "x=?" kérésre... Sőt! Egyáltalán nem tudok semmit kihozni x-re. Ha valaki segítene, annak nagyon örülnék.

Előre is nagyon köszönöm.

Ha valaki segítene, de hirtelen nem jut eszébe az egyenlet, annak mellékeltem ábraként. (A körvonal bármely P(x;y) pontja C(u;v) középponttól adott r távolságra van.) (Csakhogy precízek legyünk :)

Mégegyszer köszönöm.

[327] Iván882006-01-06 15:20:39

Ez van. Ez a sejtés nem jött be. (Közben én is rájöttem, a koszinusz-tételből) Pedid olyan szépen hangzott :-(

A matemetikus halála az, amikor egy szép sejtést tönkretesz egy csúnya tény.

Előzmény: [326] jonas, 2006-01-06 13:28:16
[326] jonas2006-01-06 13:28:16

Nem igaz.

Egyrészt azért, mert ha a\neb, akkor a paralellogrammát lapítva az átlók által bezárt szög is akármilyen lapos lehet.

Másrészt ha a paralellogramma két szomszédos csúcsát rögzíted, és a másik kettőt mozgatod (úgy, hogy az oldalhosszak rögzítettek), akkor a középpont egy körön mozog. Az átlók szöge akkor lenne állandó, ha ez a kör a rögzített oldal egy látóköre lenne, de nem az, mert a középpont nem megy a rögzített csúcsok közelébe.

Előzmény: [325] Iván88, 2006-01-05 15:58:13
[325] Iván882006-01-05 15:58:13

Igaz e, hogy az adott a, b oldalú paralelogrammákban (nem rombusz) az átlók által bezárt szög állandó? (a, b rögzített érték, az oldalak szögét változtatjuk.)

[324] lorantfy2005-12-04 10:43:37

Karl Wilhelm Feuerbach (1800-1834) Egyszerű bizonyítás a Sulineten: biz

Előzmény: [323] philip, 2005-12-03 22:37:32
[323] philip2005-12-03 22:37:32

(Fauerbach)

[322] philip2005-12-03 22:25:32

Hogyan bizonyítjuk a Fauerback-kört (kilencpont-köre)?

[321] Róbert Gida2005-11-30 20:30:07

63. feladat

Adott a síkon egy kör, melyek azok a legkisebb területű ellipszisek, melyek tartalmazzák a félkört? Igaz-e hogy az ellipszis területe kisebb, mint a kör területe?

Az egyszerűsítések érdekében feltehetjük, hogy a kör és az ellipszis középpontja ugyanazon a koordináta-tengelyen van és az ellipszis megfelelő tengelyei párhuzamosak a koordináta-tengelyekkel.

Ugyanez a kérdés magasabb ( n ) dimenzióban. Melyek azok az n dimenziós ellipszoidok melyek tartalmazzák az n dimenziós félgömböt, úgy hogy az ellipszoid térfogata minimális legyen, ez a térfogat kisebb-e mint a gömb térfogata? Ugyanazon egyszerűsítéseket most is feltehetjük mint előbb a tengelyekre vonatkozóan.

[320] lorantfy2005-11-30 09:37:41

Lehet vektorokkal is. legyen \vec {AB}= \vec x és \vec {AD}= \vec y Előállítod \vec x és \vec y segítségével az A pontból a BF felezőpontjába mutató \vec{AM}vektort és az A pontból az AE negyedelőpontjába mutató \vec{AN} vektort. Ezek egyenlőek lesznek.

Előzmény: [318] philip, 2005-11-29 19:51:23
[319] lorantfy2005-11-30 09:14:39

A legprimitívebb módszer, hogy párhuzamosokat húzunk a felező és harmadoló pontokból az oldalakkal. AE felezőpontja H. Aztán BEFH paralelogramma átlói felezik egymást.

Előzmény: [318] philip, 2005-11-29 19:51:23
[318] philip2005-11-29 19:51:23

Köszönöm szépen a segítséget! Lenne mégegy feladatom: Az ABCD négyszög paralelogramma,amelynek BC oldalát az E pont harmadolja úgy,hogy E C-hez van közelebb,az F pedig a DC oldalt felezi.Bizonyítsuk be,hogy a BF szakasz az AE szakaszt negyedeli,az AE a BF szakaszt felezi!

[317] jonas2005-11-24 10:11:26

Hmm. A 18 tényleg rossz.

Akkor x=1/(1/b+1/c)=8.

Ez onnan jön ki, hogy az FEC és a DBE háromszög hasonló az ABC-hez, mert az oldalaik párhuzamosak, így aztán a CE szakasz ax/c, az EB szakasz ax/b, amiből a=ax/c+ax/b.

Előzmény: [316] lorantfy, 2005-11-24 08:11:52
[316] lorantfy2005-11-24 08:11:52

Szia Jónás!

Te hogy értelmezted? Mert nem tudom hogy lehet 18 a rombusz oldala?

\frac{x}{12}=\frac{24-x}{24}Amiből x=8.

Előzmény: [315] jonas, 2005-11-23 20:57:43
[315] jonas2005-11-23 20:57:43

Mi a kérdés?

 \frac{b + c}2 = 18 a rombusz oldalának hossza, azt hiszem. 46.56 fokos a szöge, ha el nem számoltam.

Előzmény: [314] philip, 2005-11-23 17:43:45
[314] philip2005-11-23 17:43:45

Sziasztok! Az alábbi feladat megoldásában szeretnék segítséget kérni:

1.Egy háromszög oldalainak hossza c=24 a=18 b=12.Írjunk bele olyan rombuszt,amelynek egyik csúcsa az A,a többi csúcsa a háromszög oldalaira illeszkedik.

Eéőre is köszönöm!

[313] nadorp2005-11-22 12:09:49

Azt hiszem van valami a szimplexre. Először belátunk egy állítást:

Legyenek a,b,c pozitív számok, min(a,b,c)=a, b\neqc Ekkor léteznek olyan x,y,z pozitív számok,hogy x<a,

x+y+z=a+b+c

x2+y2+z2=a2+b2+c2,továbbá xyz<abc is teljesül. Biz:

Az egyenletrendszer ekvivalens az alábbival ( ):

y+z=a+b+c-x

y2+z2=a2+b2+c2-x2,azaz

yz=x2-(a+b+c)x+(ab+ac+bc).

Egy kis számolással adódik, hogy a fenti egyenletrendszer olyan másodfokú egyenletre vezet, melynek diszkriminánsa b\neqc esetén alkalmas x<a-ra pozitív,ezért a fenti egyenletrendszernek létezik ezzel az x-szel y,z pozitív megoldása.Már csak az abc>xyz egyenlőtlenséget kell belátni.

xyz=x3-(a+b+c)x2+(ab+ac+bc)x=f(x). Mivel f(0)=0 és f(a)=abc, ezért ha bebizonyítjuk, hogy f(x) a [0,a]-n monoton, akkor kész vagyunk. Ehhez elég belátni, hogy a derivált függvény gyökei nagyobb egyenlőek, mint a. Ez egy kis számolással a (b-a)(c-a)\geq0 nyilvánvaló egyenlőtlenségre vezet.

Az eredeti feladat ezek után egyszerű. Ha

\sum_{k=1}^nx_k=n és

\sum_{k=1}^nx_k^2=\frac{n(n-1+\alpha^2)}{n-1}, akkor

\frac{n-x_n}{n-1}=\frac{\sum_{k=1}^{n-1}x_k}{n-1}\leq\sqrt{\frac{\sum_{k=1}^{n-1}x_k^2}{n-1}}=\frac{\sqrt{n(n-1+\alpha^2)-(n-1)x_n^2}}{n-1},ebből

xn\geq1-\alpha. A fenti állításból következik, hogy a koordináták szorzata akkor minimális, ha valamelyik xi éppen 1-\alpha, a többi pedig egyenlő.Ez viszont nem lehet más,mint 1+\frac\alpha{n-1}

[312] Lóczi Lajos2005-11-21 20:35:48

Eddig mi már beláttuk az a.) pontot, sőt annál többet is: a szorzat a minimumot/maximumot olyan pontokban veszi fel (és nem csak "veheti"), ahol az xi számok pontosan 2 különböző értéket vesznek fel, l. a Lagrange multiplikátoros (valamint kompaktsági) érvelést.

A b.) rész n=3-ra a [302]-es hozzászólás-beli utolsó képletemből szinte triviális, két polinomot kell összehasonlítani, a k=2 érték minden \alpha>0 esetén kisebb, mint a k=1-hez tartozó.

Hiányzik a c.) rész, valamiféle induktív érvelés kellene tehát. Ha ez meglenne, akkor

ezek alapján a d.) kérdésre is pozitívan válaszoltunk.

Mindenesetre rögtön adódott egy szép és nehéz egyenlőtlenség a [302]-es végéről. Ha n nagy, akkor annak a függvénynek a képe "kotangens" jellegű, ilyen típusú függvényekről pedig tudom, hogy meggyűlt velük már a bajom :)

Előzmény: [311] Róbert Gida, 2005-11-21 19:49:09
[311] Róbert Gida2005-11-21 19:49:09

Nem úgy látom be, segítség a 61. feladathoz:

62. feladat

a. Legyen a és b rögzített pozitív valós számok és n>1 pozitív egész, ahol \frac{n^2}{n-1}*a^2>b>n*a^2. Legyen \sum _{i=1}^n x_i=n*a és \sum _{i=1}^n x_{i}^2=b ,x=(x1,x2,...,xn)\geq0 , akkor \prod _{i=1}^n x_i a minimumot olyan pontban veheti csak fel, ahol az xi-k pontosan két különböző értéket vesznek fel.

b. Bizonyítsuk be, hogy n=3-ra a minimumot olyan pontban veszi fel, ahol 2 darab xi megegyezik, a külöböző pedig kisebb.

c. Bizonyítsuk be n>3-ra, hogy a minimumot olyan pontban veszi fel, ahol (n-1) darab xi megegyezik, a különböző pedig ettől kisebb.

d. Ezzel bebizonyítottuk-e a 61. feladatot?! Ha igen akkor miért?

Előzmény: [310] Lóczi Lajos, 2005-11-21 13:08:17
[310] Lóczi Lajos2005-11-21 13:08:17

Legalább már "látjuk" a kontextust, de ezzel nem jutottunk közelebb a megoldáshoz :)

Azt kérdezném még, hogy be tudnád-e látni a [302]-es hozzászólás végén szereplő kifejezésről, hogy k-ban szigorúan monoton fogy (vagy várom a másik utat, hogy anélkül hogyan tudjuk minimalizálni a szorzatot...)

Előzmény: [309] Róbert Gida, 2005-11-21 07:11:15
[309] Róbert Gida2005-11-21 07:11:15

Ez a lineáris programozási feladatnak Karmarkar féle projektív módszeréhez kell ez az állítás. Konkrét tétel ( neve is van ) ez az állítás a projektív módszernél. Projektív módszer egy polinomiális futás idejű algoritmus, ellentétben a szimplex módszerrel a lineáris programozási feladatokra. Módszer lényege, hogy trafókkal eléri, hogy min(x1) ahol Ax=0,\vec {11}^t*x=n,x\geq 0-t kell meghatározni, tehát pont a mi szimplexünkőn dolgozik, egy hipersíkkal elmetszve, ráadásul úgy, hogy egy lépésben ezt az \alpha*r sugarú gömböt is használva egy kisebb x1 értékkel rendelkező megengedett megoldást talál és úgy transzformálja a feladatot, hogy újra ebbe a szimplexbe viszi a feladatot ( az A más lesz ) a megengedett megoldást pedig a gömb középpontjába, ami a csupaegy vektor.

Ha kell akkor tovább bontom a feladatot.

Előzmény: [308] Lóczi Lajos, 2005-11-20 19:34:22
[308] Lóczi Lajos2005-11-20 19:34:22

Az extremizálandó xi-szorzatnak szerintem az az érdekessége (és ez ellentmond a szemléletnek), hogy éppen akkor lesz minimális, ha a lehető legtöbb szám egynél nagyobb benne és csak 1 db lesz 1-nél kisebb; illetve akkor maximális az értéke, ha (n-1) db kisebb 1-nél és csak 1 db nagyobb 1-nél közülük.

Majd megkérdezzük Róbert Gidát, hogy honnan szedte ezt a feladatot, biztosan nem "csak úgy" kitalálta :)

Előzmény: [307] nadorp, 2005-11-20 18:59:58
[307] nadorp2005-11-20 18:59:58

Nyert,süllyedt. Ezt igencsak elnéztem.

Előzmény: [305] Lóczi Lajos, 2005-11-20 18:37:23
[306] Lóczi Lajos2005-11-20 18:53:07

[A Lagrange-multiplikátoros módszerhez a teljesség kedvéért hozzá kell tenni, hogy ugye ott lehetnek szélsőértékpontok, ahol a célfüggvény és a feltételi függvények deriváltjainak (multiplikátorokkal vett) lineáris kombinációja a nullvektor, VAGY OTT, ahol a feltételi függvények deriváltvektorai lineárisan összefüggenek. Nem egy tankönyvet láttam már, ahol ez utóbbi eshetőséget elfelejtették a példamegoldások során külön megvizsgálni (és így bizonyos, erre kihegyezett példákat nem is jól oldanának meg). A mi esetünkben ez az eset azért nem fordulhat elő, mert ahol lineáris függőség van, azok a pontok nincsenek a hipergömb felületén.

Aztán a másik apróság, hogy azt mondtuk: az xi-k egy másodfokú egyenlet megoldásai. Azonban a másodfokú egyenlet csak elsőfokú, ha az egyik multiplikátor éppen nulla lenne. Ekkor minden xi azonos lenne, de ez is ellentmondást adna a feltételi egyenletekkel.]

Előzmény: [305] Lóczi Lajos, 2005-11-20 18:37:23
[305] Lóczi Lajos2005-11-20 18:37:23

Na igen, kétféle értéket vehetnek fel az xi-k, ez igaz, az egyik értéket vegye fel k darab, a másik értéket (n-k) db (k=1,2,3,...,n-1; mind egyforma nem lehet, ezt könnyű látni). Ezeket behelyettesítve a két eredeti feltételi egyenletünkbe (t.i. az xi pontok a hipergömbfelületen és a hipersíkon is rajta vannak) megkapjuk, mi is lehet az a kétféle érték, ez az amit Te u-val és v-vel jelöltél.

Azonban u és v is függ k-tól! Tehát az Általad leírt egyenlőtlenséglánc egyáltalán nem látszik, és ez az a pont, ahol nekem is csak numerikus kísérleteim vannak.

Előzmény: [304] nadorp, 2005-11-20 17:31:44
[304] nadorp2005-11-20 17:31:44

Hülyeséget írtam, nem xi=\pmxj , hanem arra gondoltam, amire Te,hogy ebből némi esetszétválasztással kétféle érték adódhat az xikre. A vége változatlan.

Előzmény: [303] nadorp, 2005-11-20 17:17:53
[303] nadorp2005-11-20 17:17:53

Én ezt csináltam,de találtam valamit. Ha a Lagrange multiplikátor módszerrel dolgozol, akkor pld:

x1x2..xn-1=2axn+b. Mivel xn>0,ezért szorozhatunk vele

x1...xn=2axn2+bxn, azaz

2axi2+bxi=2axj2+bxj, amiből következik, hogy xi=\pmxj. Tehát a számok csak kétféle értéket vehetnek fel.Legyen ez a két érték u és v. Ekkor ha pld. u>v, akkor

uvn-1<u2vn-2<u3vn-3<...<un-1v miatt szélsőérték csak ott lehet, ahol az egyik szám egyszer, másik (n-1)-szer szerepel.

Előzmény: [302] Lóczi Lajos, 2005-11-20 15:25:56
[302] Lóczi Lajos2005-11-20 15:25:56

A pontosság kedvéért hozzá kell tennem, hogy az én bizonyításom sajnos nem teljes.

Lagrange-multiplikátorokkal némi meggondolás (és esetszétválasztás) után kiadódnak azok a pontok, ahol a szorzatnak szélsőértéke lehet: k db xi azonos az \left(1+\frac{\alpha}{\sqrt{n-1}}\cdot \sqrt{\frac{n-k}{k}}\right) számmal és (n-k) db azonos az \left(1-\frac{\alpha}{\sqrt{n-1}}\cdot \sqrt{\frac{k}{n-k}}\right) értékkel, ahol k=1,2,...,n-1.

A gömb azonban kompakt és a függvény folytonos, ezért létezik minimum és maximum, tehát a fenti pontok között ott van a szorzat minimuma és maximuma.

A numerikus kísérletek szerint k=1-nél lesz a maximum, és k=n-1-nél a minimum, de ezt nem tudtam pár óra alatt belátni (több időt pedig nem tudok rászánni egyelőre). Elég lenne bebizonyítani, hogy a

k\mapsto \left(1+\frac{\alpha}{\sqrt{n-1}}\cdot \sqrt{\frac{n-k}{k}}\right)^k \left(1-\frac{\alpha}{\sqrt{n-1}}\cdot \sqrt{\frac{k}{n-k}}\right)^{n-k}

függvény monoton fogyó a lehetséges k értékekre, a lehetséges n és \alpha paraméterértékek mellett, de ez így nem tűnik egyszerűnek.

Te milyen módszerrel jutottál túl ezen a ponton (ha egyáltalán ilyesféleképp csináltad)?

Előzmény: [301] nadorp, 2005-11-20 13:34:19
[301] nadorp2005-11-20 13:34:19

Én is erre jutottam, szélsőérték vizsgálattal. Az elemibb levezetésre egyelőre csak ötletem van.

Előzmény: [300] Lóczi Lajos, 2005-11-19 20:27:52
[300] Lóczi Lajos2005-11-19 20:27:52

Nyilván csak 0<\alpha<1 vizsgálata szükséges.

Láttuk, hogy \alpha=1 esetén valamelyik xi-nek 0-nak kell lennie, hogy a szorzat minimális legyen. Az is leolvasható az adott bizonyításból, hogy ha \alpha>0, akkor minden xi>0 kell legyen, hogy a gömbfelületen maradjunk.

A megoldás a 61. feladatra az lesz, hogy az adott gömbfelületen elhelyezkedő xi számok (i=1,2,...,n) szorzata pontosan akkor minimális, ha közülük valamely (n-1) db egyenlő az 1+\frac{\alpha}{n-1} számmal, továbbá 1 darab közülük egyenlő (1-\alpha)-val. A minimum értéke ebből már meghatározható.

Másrészt, ez ugyan nem volt kérdés, de az xi számok (i=1,2,...,n) szorzata pontosan akkor maximális, ha közülük valamely (n-1) db egyenlő az 1-\frac{\alpha}{n-1} számmal, továbbá 1 darab egyenlő (1+\alpha)-val.

Előzmény: [299] Róbert Gida, 2005-11-18 15:54:44
[299] Róbert Gida2005-11-18 15:54:44

61. feladat

Szép megoldás volt nadorp. Most jöjjön az igazi feladat. Legyen r az 59. feladatbeli beírt gömb sugara és 0<\alpha\leq1. Mennyi min\prod _{i=1}^n x_i ahol x=(x1,x2,...,xn) az a B_{\alpha r} ({\vec {11}}) felszínén van és a minimumot mely pontokban veszi fel? Ez \alpha=1-re az 59.feladat d része volt.

[298] nadorp2005-11-18 09:19:59

a) Legyen e1=(n,0,0,...,0),e2=(0,n,0,...,0)...,en=(0,0,...,n). Ekkor az ei pontok nyilván egy szabályos (n-1) dimenziós S szimplexet határoznak meg. Ha x=(x1,x2,...,xn) eleme ennek a szimplexnek, akkor

x=\sum_{i=1}^{n}\lambda_ie_i=(n\lambda_1,n\lambda_2,...,n\lambda_n), ahol \sum_{i=1}^{n}\lambda_i=1,\lambda_i\geq0, azaz \sum_{i=1}^{n}x_i=n és xi\geq0. Fordítva, ha

\sum_{i=1}^{n}x_i=n ,xi\geq0, akkor (x_1,x_2,...,x_n)=\sum_{i=1}^{n}\frac{x_i}ne_i és itt \sum_{i=1}^{n}\frac{x_i}n=1, azaz x eleme S-nek. Tehát a megoldáshalmaz egybeesik S-sel.

b) Ha s a súlypont, akkor \sum_{i=1}^{n}(s-e_i)=0, azaz, s=\frac1n\sum_{i=1}^{n}e_i=(1,1,...,1)=\vec{11}

c)Az n-dimenziós térben egy (n-1) dimenziós gömböt úgy kaphatunk,ha egy n-dimenziós gömböt elmetszünk egy (n-1) dimenzós síkkal. Ha P=(p1,p2,...,pn) a beírt gömb sugara, akkor szükségképpen S síkjában helyezkedik el,azaz \sum_{i=1}^{n}p_i=n. Határozzuk meg P távolságát a szimplex (n-2) dimenziós lapjaitól, vegyük pld. a \sum_{i=1}^{n-1}x_i=n lapot. Ekkor a (p1,...,pn-1,0) pontnak a laptól való távolsága \left|{\frac{p_1+...+p_{n-1}-n}{\sqrt{n-1}}}\right|=\frac{p_n}{\sqrt{n-1}}. Tehát P távolsága a laptól \sqrt{\frac{p_n^2}{n-1}+p_n^2}=p_n\sqrt{\frac{n}{n-1}}. Ezt elvégezve az összes többi (n-2) dimenziós lapra és felhasználva, hogy ezek a távolságok egyenlőek kapjuk, hogy p1=p2=...=pn=1, azaz P=\vec{11} és a beírt gömb sugara \sqrt{\frac{n}{n-1}}

d) Nyilván, ha xi=0, akkor a szorzat 0, azaz minimális lehet.Megmutatjuk, hogy vannak ilyen pontok. Azt kell belátni, hogy pld. létezik x=(x1,x2,...,xn-1,0) pont úgy, hogy

\sum_{i=1}^{n-1}x_i=n , xn=0 és

\sum_{i=1}^{n}(x_i-1)^2=\frac{n}{n-1}.

A második egyenlőséget átrendezve felhasználva az elsőt:

\sum_{i=1}^{n-1}x_i^2=\frac{n^2}{n-1}. Tehát

\frac{n}{n-1}=\frac{\sum_{i=1}^{n-1}x_i}{n-1}\leq\sqrt{\frac{\sum_{i=1}^{n-1}x_i^2}{n-1}}=\frac{n}{n-1}. Egyenlőség van, azaz x_1=x_2=...=x_{n-1}=\frac{n}{n-1}. Hasonlóan kapjuk a többi n-1 darab pontot is.

Előzmény: [296] Róbert Gida, 2005-11-16 22:17:53
[297] Róbert Gida2005-11-16 23:30:48

60. feladat

Bizonyítsuk be vagy cáfoljuk meg:

A szabályos 30-szög belsejében pontosan 331 darab különböző olyan pont van, amin legalább 5 átló megy át.

[296] Róbert Gida2005-11-16 22:17:53

59. feladat

Legyen \vec {11} az n dimenziós csupa 1 oszlopvektor. S legyen az \vec {11}^t*\vec x=n, \vec x\geq \vec {0} megoldáshalmaza. Bizonyítsuk be, hogy:

a: Az S egy szabályos (n-1) dimenziós szimplex

b: Mi az S súlypontja?

c: Mi az S-be írható (n-1) dimenziós gömb középpontja és sugara?

d: Mennyi min \prod _{i=1}^n x_i ahol x=(x1,x2,...,xn) a beírható gömb felszínén van. A minimumot mely pontokban veszi fel?

[295] lorantfy2005-11-15 21:02:49

Szép volt fiúk! Kösz a megoldásokat! Én is felteszek egy rajzos megoldást. Racionális osztásarányra általánosítható:-)

Előzmény: [294] nadorp, 2005-11-15 15:01:25
[294] nadorp2005-11-15 15:01:25

Jelentkező hiányában megróbálom. Húzzunk párhuzamost H1-ből CH3-mmal, ez az AB oldalt P-ben metszi.Ekkor ha PB=x, akkor a párhuzamos szelők tétele szerint H3P=2x és mivel H3 harmadoló pont, AH_3=\frac{H_3B}2=\frac32x. Az AH_3D_{\Delta} és APH_{1\Delta} háromszögek hasonlóak, a hasonlóság aránya az előbbiek szerint \frac{\frac32x}{\frac32x+2x}=\frac37, így az AH3 és AP oldalakhoz tartozó magasságok aránya is ennyi.Ha m jelöli az AP-hez tarozó magasságot, akkor azt kaptuk, hogy

\frac{T_{AH_3D}}{T_{ABH_1}} =\frac{\frac32x\frac37m}{\frac92xm}=\frac17, azaz ha T jelöli az ABC háromszög területét, akkor T_{AH_3D}=\frac{T}{21}. Hasonlóan ugyanez igaz a másik két kis háromszögre is. Másrészt ennek alapján T_{H_3BED}=\frac{T}3-\frac{2}{21}T=\frac5{21}T és ugyanez igaz a másik két keletkező négyszögre is. Összefoglalva

T_{DEF}=T-\frac3{21}T-\frac{15}{21}T=\frac{T}7

Előzmény: [290] lorantfy, 2005-11-12 22:07:51
[293] Ali2005-11-15 13:56:50

\vec AD := \mu \vec AH_1

\vec DC := \nu \vec H_{3}C

\vec AC = \vec AD + \vec DC = \mu \vec AH_1 + \nu \vec H_{3}C = \mu(\frac{1}{3}(\vec AC - \vec AB) + \vec AB) + \nu(\vec AC - \frac{1}{3}\vec AB)

\frac{\mu}{3} + \nu = 1

\frac{2\mu}{3} - \frac{\nu}{3} = 0

\mu = \frac{3}{7} ; \nu = \frac{6}{7}

AZ ADC háromszög AC oldalhoz tartozó magassága \frac{2}{3}\mu -szöröse az ABC háromszög AC oldalhoz tartozó magasságának, ezért területe is \frac{2}{3}\mu -szöröse az eredeti háromszög területének, vagyis T_{ADC} = \frac{2}{7}T_{ABC}. Ugyanez elmondható a TBFC és TAEB -ről. Következésképpen T_{DEF} = \frac{1}{7}T_{ABC}

Általában ha 1/3 helyett \sigma az oldalak felosztásának aránya, akkor T_{DEF} = (4-\frac{3}{\sigma^2-\sigma +1}) T_{ABC}

Előzmény: [290] lorantfy, 2005-11-12 22:07:51
[292] lorantfy2005-11-15 10:15:59

Egy ábra a 58. feladat -hoz:

Előzmény: [291] BohnerGéza, 2005-11-14 23:12:30
[291] BohnerGéza2005-11-14 23:12:30

58. feladat:

Az ABC háromszög beírt körének érintési pontjai A', B' és C', középpontja O, K az A'B'C' körülírt körének kp-ja.

Jelölje Ma az AC'B', Mb a BA'C' és Mc a CB'A' háromszög magasságpontját, valamint M az MaMbMc háromszög magasságpontját.

Legyen Oa az AC'B', Ob a BA'C' és Oc a CB'A' háromszög beírt körének középpontja, és K' az OaObOc körülírt körének kp-ja.

Bizonyítandó, hogy az O, a K, az M és a K' egybeesnek!

[290] lorantfy2005-11-12 22:07:51

A 46. feladat-ot még nem csinálta meg senki:

Az ABC\Delta csúcsait kössük össze a szemközti oldalak egyik harmadoló pontjával (az ábra szerint). Mekkora a keletkező DEF\Delta területe?

[289] lorantfy2005-11-08 22:14:48

A komplex számos megoldást itt a Fórumon is megtalálhatod a Felmerülő kérdések... témában Sirpi [143] hozzászólása. Ahogy az a megoldás, ez sem használja fel, hogy a szabályos háromszögek nem közös csúcsai egy körön vannak. Tehát elegendő annyit feltenni, hogy van egy közös csúcsuk.

Előzmény: [288] AzO, 2005-11-08 17:38:22
[288] AzO2005-11-08 17:38:22

Mostanaban kaptuk ezt a mackosajtos feladatot algebran, es komplex szamokkal (egyseggyokokkel) oldottuk meg, es mondta a tanar, hogy nem is akar belegondolni milyen nehez lenne elemi (kozepiskolai) modszerekkel megoldani :). Ennek ellenere ez frappans volt, es sztem megmutatom neki :) Koszi

Előzmény: [287] lorantfy, 2005-11-06 12:04:50
[287] lorantfy2005-11-06 12:04:50

Tomszy feladata a Felmerülő kérdések ... témából:

Egy r sugarú körben vegyünk fel három r sugarú húrt: AA', BB', CC'. Jelöljük A'B húr felezőpontját E-vel, B'C húr felezőpontját F-vel és C'A húr felezőpontját G-vel.

Bbh. EFG háromszög szabályos!

Megoldás vektorokkal: (Ugyanúgy megy mint a B.3837. feladat megoldása, amit a Lejárt határidejű ... témába írtam be.)

Legyenek A1,A2,B1,B2,C1,C2 a megfelelő, sugás hosszúságú szakaszok felezőpontjai.

A háromszög középvonalának tulajdonságai és a szabályos háromszögek miatt az ábrán azonos színnel jelölt vektorok egymásnak 60 fokos elforgatottjai.

\vec FE= \vec {FB_1}+\vec {B_1B_2}+ \vec {B_2E} \quad \quad
\vec FG= \vec {FC_1}+\vec {C_1C_2}+ \vec {C_2G}

Mivel a megfelelő összetevő vektorok egymás 60 fokos elforgatottjai, így \vec {FE} is 60 fokos elforgatottja \vec {FG}-nek, tehát EFG\Delta szabályos háromszög.

[286] hobbymatekos2005-09-22 21:56:39

A primszámtételből (számelméletből) másképp gondolkodni a logaritmussal kapcsolatban szimpatikus gondolat számomra.

Előzmény: [284] Fálesz Mihály, 2005-09-20 11:35:29
[285] Lóczi Lajos2005-09-20 13:59:13

Jó, de nézd meg, hogy mi a könyv célja: könnyítést adni azoknak, akiknek nehezen megy, bizonyos területekre így kevesebb előismerettel is el lehet jutni -- persze onnan nem lehet olyan messzire továbbhaladni.

(Az elvi célja pedig inkább az, ahogy írja, hogy közben a geometriát nem kell a valós számok bonyolult fogalmára építeni: tetszőleges algebrai test is használható.)

Előzmény: [284] Fálesz Mihály, 2005-09-20 11:35:29
[284] Fálesz Mihály2005-09-20 11:35:29

Én nem lelkesedek érte, sőt.

Trigonometria-feladatokban a szögeknek legfeljebb csak a szinuszát/koszinuszát szoktuk kiszámolni, magukat a szögeket nem. Négyzetgyökökkel pedig így is, úgy is számolni kell. Például elég megkérdezni, hogy ha az A,B,C pontok egy egyenesen vannak, mondjuk AB2=2 és BC2=3, akkor mekkora lehet AC2. A konstrukció csak a legegyszerűbb esetekben teszi félre a négyzetgyökvonást, a számolás egyáltalán nem lesz tőle sem könnyebb, sem egyszerűbb.

A trigonometrikus függvények elvetését pedig kb. olyan ötletnek tartom, mint ha valaki a prímszámtételből ki akarná irtani a logaritmust (mert hát ugye mi szükség van transzcendens függvényekre, ha egész számokról akarunk beszélni), vagy a Cardano-képletből a komplex számokat. Pont a matematikát akarja kiirtani; azt, hogy hozzunk létre elméleteket, új objektumokat (jelen esetben a trigonometrikus függvényeket), amik megmutatják, hogy a dolgok mögött milyen mélyebb összefüggések vannak.

F.M.

Előzmény: [282] Lóczi Lajos, 2005-09-19 22:50:10
[282] Lóczi Lajos2005-09-19 22:50:10

Szép és elegáns a leírás, sokmindenben egyetértek vele, bár nem hiszem, hogy túlzottan nagy változást okozna mifelénk belátható időn belül, ha egy olyan alapvető fogalomhoz, mint a "szög" akar hozzányúlni, átdefiniálni.

Előzmény: [281] 2501, 2005-09-19 10:25:15
[283] qer2005-09-19 11:15:16

http://web.maths.unsw.edu.au/~norman/papers/Chapter1.pdf

itt található az első fejezet a könyvéből (ez ingyen elérhető)...

Előzmény: [281] 2501, 2005-09-19 10:25:15
[281] 25012005-09-19 10:25:15

Sziasztok!

Ezt lattatok mar?

[280] Szalkai István2005-09-14 13:42:39

Kedves Mindenki !

A 2005.jan. feladatot általánosítva jutott eszembe a a következő probléma: Mi azon pontok mértani helye, amelyeknek adott egyenesektől való távolságai kielégítenek egy lineáris összefüggést? "Felfedezésem" biztosan nem új, bár irodalomban nem akadtam a nyomára. Közzéteszem mégis, hátha esetleg valakinek hasznára válik, no és biztosan én is tanulok hozzászólásaitokból! (Más fajta TEX-et használva nem sikerült a szöveget ide feltennem, de a következő linken megtalálható:

http://www.szt.vein.hu/~szalkai/Tavolsagok-jav.pdf

Üdvözlettel: szalkai@almos.vein.hu

[279] BohnerGéza2005-08-27 00:19:34

Jogos [275] jonas észrevétele, bár csak egy kis fogalmazási hibát követtem el. A zárójel szövege helyesen: valamint a talpponti háromszög területének. Igaz ez már nem nehéz, ha az előzőekre megvan a válasz.

Előzmény: [275] jonas, 2005-08-23 22:40:10
[278] lorantfy2005-08-24 10:25:21

Hello Viktor!

Ez az EUKLIDES program. A 2.02 verzió ingyenesen letölthető: www.euklides.hu.

Töltsd le és szórakozz el vele. Én a kész ábrát Print Screen-el vágólapra szoktam tenni. Aztán egy képszerkesztővek kivágom a lényeges részt és átlátszóvá teszem, majd elmentem gif-ben és úgy csatolom a hozzászóláshoz. Így a háttérszínen jelenik meg.

Előzmény: [277] xviktor, 2005-08-24 10:09:36
[277] xviktor2005-08-24 10:09:36

Ilyen abrat milyen programmal lehet csinalni?

Előzmény: [276] lorantfy, 2005-08-24 09:59:41
[276] lorantfy2005-08-24 09:59:41

Nem hiszem, hogy Géza itt a talpponti \Delta-re gondolt! Szerintem a=13, b=14, c=15 és a TCBTA\Delta-ről van szó.

Előzmény: [275] jonas, 2005-08-23 22:40:10
[275] jonas2005-08-23 22:40:10

A talpponti háromszög nem a TcTbTa?

Előzmény: [274] BohnerGéza, 2005-08-23 22:08:14
[274] BohnerGéza2005-08-23 22:08:14

Üdv mindenkinek!

Köszönöm Kós Ritának [273] a segítséget!

57. (számozott) feladat: Egy háromszög oldalai 13, 14 és 15. Adjuk meg a pontos értékét a

a.) beírt köre sugarának

b.) TcBTa (Ta és Tc magasságtalppontok) beírt köre sugarának

c.) TcBTa háromszög területének ( a talpponti háromszög területének )

[273] Kós Rita2005-07-26 19:31:42

A lekepezesek szorzatarol rovidebben-hosszabban Reiman Istvan konyveiben is van szo: Fejezetek az elemi geometriabol (Typotex, pici vekony, ebben biztosan), ill. A geometria es hatarteruletei c. konyvben, ha jol emlekszem.

Előzmény: [272] BohnerGéza, 2005-07-15 23:09:11
[272] BohnerGéza2005-07-15 23:09:11

Az 50., 51., 53. és 56. feladat közös, általános megoldása.

Igen örülök lorantfy és Kós Géza 56. feladatra a [267]-ben ill. [268]-ban leírt megoldásának. Ezek is alkalmasak az általánosításra.

Ha valaki még nem ismeri a leképezések szorzatát, annak is megérthető amit írok, de időt és energiát kell rá szánnia, végigjátszva-gondolva minden állítást!

A téma bővebb megismeréséhez Rácz János könyveit tudom ajánlani, de ezek nehezen érhetők el. Rossz memóriám miatt további könyveket most nem tudok, remélem lesz valaki és kisegít!

Jelölje az A körüli alfa forgatást (A|alfa). Legyen

(1)...(C|gamma)*(B|béta)*(A|alfa)=I helybenhagyás.

//A leképezések szorzatát - egymás utáni elvégzését - visszafelé olvasva kell értelmezni, tehát először A, majd B, végül C körül forgatunk,// Helybenhagyást akkor kapunk, ha összesen n*360 fokot, ahol n egész, forgatunk és van fixpont. Ez a fent jelzett feladatok esetén áll. Az egyszerűség kedvéért és mert ilyen esetre ezen feladatok mindig visszavazethetők n=1 (vagy -1) esettel foglalkozunk.

Tudnunk kell még, hogy egy forgatás helyettesíthető két tengelyes tükrözés szorzatával, pl. (A-alfa)=t2*t1, ahol t1 és t2 is átmegy A-n, valamint t1 és t2 szöge alfa fele az irányítást is figyelembe véve. (1)-et balról (C|-gamma)-val szorozva:

(2)...(B|béta)*(A|alfa)= (C|-gamma)

Legyen t2=AB, t1 és t3 pedig olyan egyenesek, melyekre (A|alfa)=t2*t1 és (B|béta)=t3*t2. Ekkor

(3)...(B|béta)*(A|alfa)=(t3*t2)*(t2*t1)=t3*(t2*t2)*t1=t3*t1= (C|-gamma)

Tehát t1 és t3 is átmegy C-n. Mindent végiggondolva ABC olyan háromszög kell legyen, melyben a megfelelő csúcsoknál alfa/2, béta/2 ill. gamma/2 szög van. //Feltéve, hogy egyik szög sem n*360 fok, azaz mind a három forgatás valódi fogatás. //

Jó munkát a fent jelzett feladatok átgondolásához! Kitalálható esetleg újabb konkrét feladat is?!

Előzmény: [268] Kós Géza, 2005-07-11 12:00:08
[271] Hajba Károly2005-07-11 16:43:28

Köszi a továbbképzés. Tényleg egyszerű.

De talán már nem olyan egyszerű azon zárt görbe megszerkesztése, mely görbe bármely pontjára azonos a PA+PB+PC hossz nagysága. Ezen görbe elfajult esete az I pont is.

HK

Előzmény: [270] lorantfy, 2005-07-11 15:40:54
[270] lorantfy2005-07-11 15:40:54

Kedves Károly!

A tétel igaz: Ha az ABC hegyesszögű háromszög síkjában lévő P pontra igaz, hogy AP+BP+CP összeg minimális, akkor P a háromszög izogonális pontja (melyből mindhárom oldal 120 fokos szögben látszik.)

A bizonyítás nagyon szép és egyszerű. Forgassuk el a B pont körül a BCP háromszöget 60 fokkal. Mivel BP'P háromszög egyenlő oldalú, ezért az APP'A' törött vonal hossza megegyezik az AP+BP+CP összeggel. Az A' helyzete P-től független. APP'A' hossza akkor minimális, ha P és P' az AA' egyenesre illeszkedik. Ez pedig akkor van, ha APB\angle és BPC\angle 120o, vagyis ha P az izogonális pont.

Előzmény: [269] Hajba Károly, 2005-07-11 13:31:23
[269] Hajba Károly2005-07-11 13:31:23

Üdv!

Érdekesnek tűnik a háromszög I pontja más szempontból is, mintha erre a pontra igaz, hogy PA + PB + PC hossz a legrövidebb, ahol A, B, C a háromszög csúcsai, míg P egy tetszőleges pont a síkjukban. Bizonyítani nem tudom, csak ráleltem. Ha igaz, gondolom egy -általam nem- ismert tétel.

Egy adtok egy kis továbbképzést vagy címeket, megköszönném. :o)

O.

Előzmény: [268] Kós Géza, 2005-07-11 12:00:08
[268] Kós Géza2005-07-11 12:00:08

Legyen az AC'B és BA'C köré írt körök B-től különböző metszéspontja I. Az AC'BI és BA'CI húrnégyszögek szögeiből AC'B\angle=BA'C\angle=120o. Ebből következik, hogy CB'A\angle=120o, vagyis az I pont a CB'A körön is rajta van.

Ha az ABC háromszög mindegyik szöge 120 foknál kisebb, akkor I a háromszög izogonális pontja. Ha valamelyik szög éppen 120 fok vagy annál nagyobb, akkor nincs izogonális pont, és az ábra kicsit máshogy néz ki, de a három kör akkor is egy ponton megy át.

Az IA'', IB'', IC'' szakaszok (egyenesek) páronként 120 fokos (60 fokos) szöget zárnak be. Az A''B'', B''C'', C''A'' egyenesek két-két kör centrálisai, amik merőlegesen felezik az IA'', IB'', IC'' közös húrokat. Ezek az egyenesek tehát szintén páronként 60 fokos szöget zárnak be egymással.

Előzmény: [267] lorantfy, 2005-07-10 17:00:12
[267] lorantfy2005-07-10 17:00:12

Nem szeretném, ha Géza szép feladata feledésbe merülne!

56. feladat megoldása: Helyezzük a koord.rsz. origóját a háromszög S súlypontjába.

Legyen

\vec{SA}=\vec{a}=(a_1;a_2)\quad
\vec{SB}=\vec{b}=(b_1;b_2)\quad
\vec{SC}=\vec{c}=(c_1;c_2)

Fejezzük ki \vec{SA"} vektort ezek segítségével!

\vec{SF_A}=(-\frac{a_1}{2};-\frac{a_2}{2})

\vec{F_AA"} pedig \vec{CB} 90 fokos, pozitív irányú elforgatottjának \frac{\sqrt{3}}{6}-szorosa.

\vec{F_AA"}=\frac{\sqrt{3}}{6}(c_2-b_2;b_1-c_1)

\vec{SA"}=\vec{SF_A}+\vec{F_AA"}=(\frac{\sqrt{3}}{6}c_2-\frac{\sqrt{3}}{6}b_2-\frac{a_1}{2};\frac{\sqrt{3}}{6}b_1-\frac{\sqrt{3}}{6}c_1-\frac{a_2}{2})

Hasonlóan:

\vec{SB"}=\vec{SF_B}+\vec{F_BA"}=(\frac{\sqrt{3}}{6}a_2-\frac{\sqrt{3}}{6}c_2-\frac{b_1}{2};\frac{\sqrt{3}}{6}c_1-\frac{\sqrt{3}}{6}a_1-\frac{b_2}{2})

Az origó körüli 120 fokos, pozitív irányú forgatás mátrixa:

\left[\matrix{cos 120^\circ &sin 120^\circ \cr -sin 120^\circ & cos 120^\circ \cr}\right]=
\left[\matrix{-\frac{1}{2}&\frac{\sqrt3}{2}\cr\frac{-\sqrt3}{2} &-\frac{1}{2} \cr}\right]

Ezzel beszorozva \vec{SA"}-t és felhasználva, hogy a,b,c vektorok összege 0, vagyis a koordinátákra is:

a1+b1+c1=0,  a2+b2+c2=0

\vec{SB"}-t kapjuk. Tehát igaza van Dánielnek (=tudniakarok): valóban egyenlő oldalú háromszöget kapunk.

[266] Lóczi Lajos2005-07-06 13:04:55

Esetleg érdemes lehet keresgélni a háromszög jelenleg 3055 nyilvántartott nevezetes pontja között

http://mathworld.wolfram.com/KimberlingCenter.html

és az itteni hivatkozásokban, különösképp

http://faculty.evansville.edu/ck6/encyclopedia/

Jó böngészést! :)

Esetleg egy geometriai rajzprogram is segíthet a kísérletezgetésben.

Előzmény: [264] papi, 2005-07-06 08:37:07
[265] papi2005-07-06 08:54:13

Bocsánat ! Természetesen az előbb Torricelli pontot akartam írni (papi)

[264] papi2005-07-06 08:37:07

Kedves Barátaim ! A h-szög nevezetes pontjainak a köré írható körre vonatkozó hatványait keresgélem. A S-pont, M-pont, beírt kör Kpontja és a Lamoine-féle pont esetében már rájöttem e hetványokra. De a Tossicelli-pontnál elakadtam. Tud valaki segíteni? (papi)

[263] Lóczi Lajos2005-07-04 23:46:42

Csak kiegészítésképpen írom, hogy a "négyzetes közelítés" az idézett Newton-módszerben persze nem azt jelenti, hogy a szereplő deriváltpolinom épp másodfokú, hanem azt, ahogyan a hiba nagysága csökken: a numerikus módszerek elméletéből ismert, hogy a Newton-módszer gyorsan konvergál, ha megfelelően közelről indítjuk a tényleges megoldástól (azaz z0-t "elég közel" választjuk meg a keresett értékhez, ami jelen esetben z0=1 mellett jó, de általában a pontos vonzási tartomány fraktálbonyolultságú alakzatokból áll, l. pl. http://mathworld.wolfram.com/NewtonsMethod.html)

A konvergencia a jelen esetben olyan gyors, hogy |z3-cos (10o)|\le2.10-7, majd |z4-cos (10o)|\le3.10-14, aztán |z5-cos (10o)|\le7.10-28, stb.

(A hatványsoros közelítés előnye, hogy a kezdeti értékkel nem kell bajlódni, de a konvergencia nem ennyire gyors: az n. lépésben a hiba ott n faktoriálissal arányos.)

Előzmény: [261] jonas, 2005-07-02 10:28:45

  [1]    [2]    [3]    [4]    [5]    [6]    [7]    [8]    [9]    [10]